Moving Charges and Magnetism Notes and Assignment
Moving Charges and Magnetism Notes and Assignment
Center – Complex no. !!2 , 1st floor , Sector 7 market , Ambala City
Contact No.- 9896441144                                                                   Page 1
        MITTAL INSTITUTE FOR PHYSICS
                        Tm         or Wbm                                           tesla
    Here μ0 is a constant called                         Thus one tesla is 107 times the
permeability of free space. So the                   magnetic field produced by a conducting
Biot-Savart law in SI units may be                   wire of length one metre and carrying
expressed as                                         current of one ampere at a distance of one
                                                     metre from it and perpendicular to it.
                                                     4.4 BIOT-SAVART              LAW          VS.
We can write the above equation as                   COULOMB'S LAW
                                                          4. Give some points of similarities and
                                                     differences between Biot-Savart law for the
    As the direction of dB is perpendicular          magnetic field and Coulomb's law for the
to the plane of ⃗⃗⃗ and , so from the above          electrostatic field.
equation, we get the vector form of the
Biot-Savart law as                                       Comparison of Biot-Savart law with
                                                     Coulomb's law. According to Coulomb's
                             ⃗⃗⃗                     law, the electric field produced by a
                ⃗⃗⃗⃗⃗
                                                     charged element at a distance r is given by
     the source and the field point. The          200 cm north-east from the piece.
     direction of the magnetic field is
                                                      Solution. The problem is illustrated in
     perpendicular to the plane containing
                                                  Fig. 4.4.
     the displacement vector     and the
     current element I ⃗⃗⃗ .
3. In Bio-Savart law, the magnitude of the
   magnetic field is proportional to the
   sine of the angle between the current
   element I ⃗⃗⃗ and displacement vector
   while there is no such angle
   dependence in the Coulomb's law for
   the electrostatic field. Along the axial                          Fig. 4.4
   line of the current element θ=0°, sinθ=0
                                                      As the distance OP is much larger than
   and hence dB = 0.
                                                  the length of the wire, we can treat the wire
5. Write a relation between μ0,    and a          as a small current element.
Relation between μ0,       and c. We know         Here       A
   that
                                                           cm
and Tm
∴
                                                                 .
Biot-Savart law,
Units Used
Magnetic field B is in tesla, current I in
ampere and distance r in metre.
Constant Used
Permeability         constant,
    Tm                                                               Fig. 4.5
                                                   HUNTS
                                                   Proceed as in Example 1.
                                 T                 Proceed as in Example 2.
    The direction of the field ⃗⃗⃗⃗⃗ will be the   Proceed as in Example 2.
direction of vector ⃗⃗⃗  . But                         We shall now apply Biot-Savart law to
  ⃗⃗⃗                                        ̂     calculate the magnetic field due to (i) a
                ̂      ̂             ̂   ̂
                                                   straight current carrying conductor and (ii)
    Hence field ⃗⃗⃗⃗⃗ is in the + z-direction.     a circular current loop.
Problems for Practice                              4.5 MAGNETIC FIELD DUE TO A LONG
                                                       STRAIGHT CURRENT CARRYING
1. A wire placed along east-west direction             CONDUCTOR
   carries a current of 10 A from west to
   east direction. Determine the magnetic              6. Apply Biot-Savart law to derive an
   field due to a 1.8 cm piece of wire at a        expression for the magnetic field produced
   point 300 cm north-east from the piece.         at a point due to the current flowing through
                                                   a straight wire of infinite length. Also draw
(Ans                T, normally out of the plane   the sketch of the magnetic field. State the
                                       of paper)   rules used for finding the direction of this
2. A small current element I ⃗⃗⃗ , with ⃗⃗⃗ = 2    magnetic field.
k mm and I = 2 A is centred at the origin.              Magnetic field due to a long straight
Find magnetic field ⃗⃗⃗⃗⃗ at the following         current carrying conductor As shown in
points :                                           Fig. 4.7, consider a straight conductor XY
                                                   carrying current I. We wish to find its
(i) On the x-axis at x = 3 m.
                                                   magnetic field at the point P whose
    (Ans. 4.44 × 10-11 ̂T)                         perpendicular distance from the wire is a
                                                   i.e., PQ = a.
(ii) On the x-axis at x = - 6 m.
(Ans.                      ̂ )
(iii) On the z-axis at z = 3 m.
(Ans. 0)
3. An element ⃗⃗⃗ =      ̂ is placed at the
   origin (as shown in Fig. 4.6) and carries
   a current I = 2 A. Find out the magnetic
   field at a point Pon the y-axis at a
   distance of 1.0 m due to the element Ax
   - 1 cm. Give also the direction of the
   field produced. [CBSE D 09C]
         (Ans.           T in      -direction)
                                                   Fig. 4.7 Magnetic field due to a straight
                                                           current carrying conductor.
                                                        Consider a small current element ⃗⃗⃗ of
                                                   the conductor at O. Its distance from Q is l
                                                   i.e., OQ = l. Let be the position vector of
                                                   point P relative to the current element and
                                                   θ be the angle between dl and r. According
                                                   to Biot-Savart law, the magnitude of the
                                                   field ⃗⃗⃗⃗⃗ due to the current element ⃗⃗⃗ will
                                                   be
Fig. 4.6
θ + ϕ =90°                                        ∴
or θ = 90° - Φ                                         Clearly, the magnetic field due to an
∴ sin θ = sin (90° - Φ) = cos Φ                   infinitely long straight current carrying
                                                  conductor at its one end is just half of that
 lso cos Φ =                                      at any point near its middle, provided the
                                                  two points are at the same perpendicular
or                                                distance from the conductor.
                                                      3. If the conductor is of finite length L
As                                                and the point P lies on its perpendicular
∴ 1 = a tan Φ                                     bisector, then
                                                                     ⁄
On differentiating, we get
                                                               √                 √
dl = a sec2 Φ dΦ
Hence
Or
                                                                         √
    According to right hand rule, the
direction of the magnetic field at the P due
to all such current elements will be in the                             √
same direction, namely, normally into the              Direction of magnetic field. For an
plane of paper. Hence the total field B at        infinitely long conductor,
the point P due to the entire conductor is
obtained by integrating the above equation
within the limits    and
          ∫              ∫
                                                       Clearly, the magnitude of the magnetic
                                                  field will be same at all points located at the
                                                  same distance from the conductor. Hence
                                                  the magnetic lines of force of a straight
     2. Maxwell's cork screw rule. If a right     Magnetic field due to a straight current
handed screw be rotated along the wire so         carrying conductor
that it advances in the direction of current,     Formulae Used
then the direction in which the thumb
rotates gives the direction of the magnetic       1. Magnetic field due to a straight
field (Fig. 4.10).                                   conductor of finite length,
                                                  Units Used
                                                  Magnetic field B is in tesla, current I in
                                                  ampere and distance α in metre.
                                                  Example 3. A current of 10 A is flowing east
                                                  to west in a long wire kept horizontally in
                                                  the east-west direction. Find magnetic field
                                                  in a horizontal plane at a distance of
   (i) 10 cm north
                                                                √
    (ii) 20 cm south from the wire; and in
the vertical plane at a distance of                         √
   (iii) 40 cm downward and                                                     T
   (iv) 50 cm upward.                             Example 5. Figure 4.12 shows two
                                                  current-carrying wires 1 and 2. Find the
    Solution, (i) Magnetic field in a
                                                  magnitudes and directions of the magnetic
horizontal plane at 10 cm north of the wire
                                                  field at points P, Q and R.
is
                                     T
    According to right hand thumb rule, the
direction of the magnetic field will be
downward in the vertical plane.
    (ii) Magnetic field at 20 cm south of the
wire is
                                         T
                                                                    Fig. 4.12
    The magnetic field will point upward in           Solution, (i) According to right hand grip
the vertical plane.                               rule, the field B1 of wire 1 at point P will
    (iii) Magnetic field 40 cm just down the      point normally outward while the field B2 of
wire is                                           wire 2 will point normally inward, hence
T p
                  Fig. 4.13                                              √               √
    Solution. At the required point, the
resultant magnetic field will be zero when        directed normally outwards.
the fields due to the two wires have equal
magnitude and opposite directions. Such           Example 8. A current of 1.0 A is flowing in
point should lie either to the left of P or to    the sides of an equilateral triangle of side
the right of Q. But the wire Q has a smaller      4.5 × 10-2 m. Find the magnetic field at the
current, the point should lie closer to and to    centroid        of      the         triangle.
the right of Q. Let this point be R at                                                       [
distance x from Q, as shown in Fig. 4.13.         Roorkee 91]
Field due to current f at point R,                   Solution. The situation is shown in Fig.
                                                  4.15. The magnetic field at the centre O
                                                  due to the current through side PQ is given
                                                  by
normally into the plane of paper. Field due
to current I2 at point R,
Or
         cm
                 cm
                                                               ∴
                                                                                     √
                                                                                         √
                                                                                             [
                                                                                 ]
                                                           √        √        √
                                                                                                   T
                  Fig. 4.14
                                                  directed normally outwards.
   The magnetic field at O due to
conductors BC, CD and DA will also be of          Example 9. Figure 4.16 shows a
same magnitude and direction. Therefore,          right-angled isosceles APQR having its
resultant field at 0 is                           base equal to a. A current of I ampere is
                                                  passing downwards along a thin straight
wire cutting the plane of paper normally as         3. The magnetic induction at a point P
shown at Q. Likewise a similar wire carries            which is at a distance of 4 cm from a
an equal current passing normally upwards              long current carrying wire is 10-3 T.
at R. Find the magnitude and direction of              What is the magnetic induction at
the magnetic induction B at P. Assume the              another point Q which is at a distance
wires to be infinitely long.                           of 12 cm from this current carrying
                                                       wire?      (Ans. 3.33 ×10-4 T)
[ISCE 97]
                                                    4. What current must flow in an infinitely
                                                       long straight wire to give a flux density
                                                       of 3 × 10-5 T at 6 cm from the wire?
                                                           (Ans. 9 A)
                                                    5. A vertical wire in which a current is
                                                       flowing produces a neutral point with
                    Fig. 4.16                          the earth's magnetic field at a distance
                                                       of 10 cm from the wire. What is the
     Solution. Let PQ = QR = r. In right
                                                       current if BH = 1.8 × 10-4 T? (Ans. 90 A)
      ,
                                                    6. Fig. 4.17 shows two long, straight wires
                                or                     carrying electric currents of 10 A each
                                         √             in opposite directions. The separation
   Magnetic induction at point P due to the            between the wires is 5.0 cm. Find the
conductor passing through Q                            magnetic field at a point P midway
                                                       between the wires.
              √
                           acting along PR
                      √                                                         (Ans. 1.6 ×10-6 T)
   Magnetic induction at point P due to the
conductor passing through R,
              acting along PQ
      √
                     √
                                                    7. Two long parallel wires are placed at a
                                                       distance of 16 cm from each other in
                                                       air. Each wire has a current of 4 A.
      √                              √
          √           √                  √             Calculate the magnetic field at midpoint
                                                       between them when the currents in
Or                                                     them are (i) in the same direction and
                                                       (ii) in opposite directions. [Ans. (i) Zero
  This field acts towards the midpoint of              (ii) 2 × 10-5 T]
QR.
                                                    8. Two infinitely long insulated wires are
Problems for Practice                                  kept perpendicular to each other. They
                                                       carry currents I1 = 2 A and I2 = 1.5 A. (i)
1. A straight wire carries a current of 3 A.
   Calculate the magnitude of the                      Find the magnitude and direction of the
                                                       magnetic field at P. (ii) If the direction of
   magnetic field at a point 10 cm away
   from the wire.     [CBSE D 96]                      current is reversed in one of the wires,
                                                       what would be the magnitude of the
                                (Ans. 6 × 10-6 T)      field B ?
2. At what distance from a long straight            [Ans. (I) 2 × 10 5 T, normally into the plane
   wire carrying a current of 12 A will the         of paper (ii) zero]
   magnetic field be equal to 3 × 10-5 Wb
   m-2. (Ans. 8 × 10-2 m)
                                                                                    [IPUEE 13]
                                                                                       √
                                                                            ( ns.                  )
                                                                                    cm       m
                                                           √                  √          √
or
                                                                                                 T
                                         T
                                                  (ii) Net current, I = 0.5 × 4 - 0.5 × 2 = 1.0 A
          [     cm               m]
                                                                                                 T
7. (i) When the currents are in same
   direction,                                     11. Refer to Fig. 4.21. Magnetic field at O
                                                      due to finite wire AB,
8. T
                                    T
For point C, θ = 0°, therefore
 s          so
Hence the net magnetic field at the point P,
                                                  or
                               (       )
                                                     If instead of a single loop, there is a coil
directed normally inward.                         of N turns, all wound over one another,
4.6 MAGNETIC FIELD AT THE CENTRE                  then
    OF CIRCULAR CURRENT LOOP
   7. Apply Biot-Savart law to derive an
expression for the magnetic field at the          4.7 MAGNETIC FIELD ON THE AXIS OF
centre of a current carrying circular loop.           A CIRCULAR CURRENT LOOP
    Magnetic field at the centre of a circular        8. Apply Biot-Savart law to find the
current loop. As shown in Fig. 4.23,              magnetic field due to a circular current
consider a circular loop of wire of radius r      carrying loop at a point on the axis of the
carrying current I. We wish to calculate its      loop. State the rules used to find the
magnetic field at the centre O. The entire        direction of this magnetic field.
circumference
                                                      or                     ⁄
                                                            ⁄
                                                                ⃗                    ̂
                                                                                 ⁄
Fig. 4.24 Magnetic field on the axis of a            As the direction of the field is along +ve
          circular current loop.                  X-direction, so we can write
    Consider a current element dl at the top
of the loop. It has an outward coming                           ⃗                    ̂
                                                                                 ⁄
current.
                                                  If the coil consists of N turns, then
    If s be the position vector of point P
relative to the element ⃗⃗⃗ , then from                         ⁄
hydrogen atom, an electron revolves                        lies in the XY plane with its centre at the
around the nucleus in a circular orbit of                  origin of co-ordinates. Find the magnetic
radius 5.11 × 10-11 m at a frequency of 6.8                field, at the point (0, 0, 20 √ cm), when
× 1015 Hz. What is the magnetic field set up               this coil carries a current of ( )
at the centre of the orbit?
                                                                    [CBSED16C]
   Solution. If n is the frequency of
revolution of the electron, then                                Solution. Here N = 100, a = 20 cm = 0.2
                                                           m
√ cm √ m
Equivalent current, ⁄
            Charge                                                              axial
             Time
                                                                                                              ⁄
                                                                            centre
Example 1 3. A circular coil, having 100                   Example 15. Two identical circular coils of
turns of wire, of radius (nearly) 20 cm each,              radius 0.1 m, each having 20 turns are
                                                           mounted co-axially 0.1 m apart. A current
of 0.5 A is passed through both of them (i)               Solution. For the net magnetic field at
in the same direction, (ii) in the opposite          the point O to be zero, the direction of
directions. Find the magnetic field at the           current in loop L2 should be opposite to that
centre of each coil.                                 in loop
Solution. Here a =0.1 m, N =20, r = 0.1 m, I              Magnitude of magnetic field due to
   =0.5 A                                            current     in    = Magnitude of magnetic
   Magnetic field at the centre of each coil         field due to current in
due to its own current is
                                                       or                            ⁄
T ⁄
                                             T
            √
                                                                         Fig. 4.28
(i) When the currents are in the same
    direction, the resultant field at the            Solution. Magnetic field due to linear
    centre of each coil is                           portion. Any element ⃗⃗⃗ of linear portions
                                                     like PQ or ST will make angles 0 or π with
                                                 =
                                                     the position vector r. Therefore, field at O
   8.50 × 10-5 T.
                                                     due to linear portion is
(ii) When the currents are in opposite
     directions, the resultant field is
                                                 =       Magnetic field due to semi-circular
4.06 × 10-5 T.
                                                     portion. Any element ⃗⃗⃗ on this portion will
Example 16. Two coaxial circular loops               be perpendicular to the position vector ,
       and               of     radii  3             therefore, field due to one such element at
                                       c             point will be
m and 4 cm are placed as shown. What
should be the magnitude and direction of
the current in the loop so that the net
magnetic field at the point O be zero?                   Magnetic field due to the entire circular
                                                     portion is given by
                             [CBSE SP 08]
∴ Total field at O is
                                                                     (        ), up the plane of
                                                  paper.
                                                  Example 20. Figure 4.31 shows a current
                  Fig. 4.29                       loop having two circular segments and
                                                  joined by two radial lines. Find the
centre of the arc ? Would your answer             magnetic field at the centre O.
change if the wire were bent into a
semicircular arc of the same radius but in
the opposite way as shown in Fig. 4.29(b)?
[NCERT; Pb 91]
    Solution, (i) Magnetic field at the centre
of the arc is
Here, I= 12 A r =2.0 cm =0.02 m, μ0 =4π×
   10-7 TmA-1
                           Tm                                       Fig. 4.31
                                                      Solution. Since the point O lies on lines
                                        T         SP and QR, so the magnetic field at O due
                                                  to these straight portions is zero.
    According to right hand rule, the
direction of the field is normally into the           The magnetic field at O due to the
plane of paper.                                   circular segment PQ is
(ii) The magnetic field will be of same
     magnitude,              T
    The direction of the field is normally out        Here, l = length of arc PQ = a
of the plane of paper.                                          directed normally upward
Example 19. A long wire is bent as shown
in Fig. 4.30. What will be the magnitude              Similarly, the magnetic field at O due to
and direction of the field at the centre O of     the circular segment SR is
the circular portion, if a current I is passed                 directed normally downward.
through the wire? Assume that the various
portions of the wire do not touch at point P.         The resultant field at O is
                  Fig. 4.32                                                                       T
                           rc                     normally out of the plane of paper.
Solution.    s   rad     Radius
                                                  Example 23. The current-loop PQRSTP
∴           or                                    formed by two circular segments of radii R1
                                                  and R2 carries a current of I ampere. Find
     According to Biot-Savart law, magnetic       the magnetic field at the common centre O.
field at the centre O is                          What will be the field if angle a =90°?
                                                     Solution. The magnetic field at O due to
                                                  each of the straight parts PQ and RS is
                                                  zero because θ = 0°, for each of them.
                              T
Example 22. In Fig. 4.33, abed is a circular
coil of non-insulated thin uniform
conductor. Conductors pa and qc are very
long straight parallel conductors tangential
to the coil at the points a and c. If a current
of 5 A enters the coil from p to a, find the
magnetic induction at O, the centre of the                         Fig. 4.34
coil. The diameter of the coil is 10 cm.              Magnetic field at the centre O due to
                                                  circular segment QR of radius R2 is
                                                  Here,
                                                  l2 = length of circular segment
                  Fig. 4.33                                   directed normally downward
    Solution. Here Iabc = Iadc =2.5 A,
                                                      Similarly, the magnetic field at O due to
                                          cm      the circular segment STP is
                                      m
                                                                    ,    directed       normally
    The magnetic induction at O due to the
current in part abc of the coil is equal and      downward
opposite to the magnetic induction due to         Hence the resultant field at O is
the current in part adc. So magnetic
induction at O due to the coil is zero.                                     (                 )
    Magnetic induction at O due to the
straight conductor pa (a half infinite            directed normally downward If           α
segment) is                                           =90° = π/2, then
                                  T                                                 [         ]
normally out of the plane of paper.               Example 24. A current I = 5.0 A flows along
a thin wire shaped as shown in Fig. 4.35.             Now suppose the wire B is bent into a
The radius of the curved part of the wire is      square of side L. We know that the
equal to R =120 mm, the angle 2 ϕ = 90°.          magnetic field due to a wire of finite length
Find the magnetic induction of the field at       whose ends make angles α and β with the
the point O.                                      perpendicular dropped on wire from the
                                                  given point at distance r from it is given by
                Fig. 4.35,
    Solution. Magnetic induction at O due
to the line segment AB is
Fig. 4.36
                       /   acting   normally
downwards
                                                      By symmetry, magnetic field at O due
Total magnetic induction at O,
                                                  to all the four sides of the square will be in
                                                  the same direction. Hence total field at O
                                                  due to the current-carrying square is
                                                            √        √
                                                                           ...(2)
                                                      Comparing equations (1) and (2), we
                                        T         find that the square wire produces a
                                                  greater field at its centre.
Example 25. Two wires A and B have the
same length equal to 44 cm and carry a                (b) Magnetic field at the centre of the
current of 10 A each. Wire A is bent into a       circular wire is
circle and wire B into a square, (a) Which
wire produces a greater magnetic field at                                                   T
the centre? (b) Obtain the magnitudes of
the fields at the centres of the two wires.                           T              cm
                          √
                 ⃗
                 √                                                                [CBSE OD 17]
Problems for Practice                                                 (Ans.       T              )
1. Consider a tightly wound 100 turn coil
   of radius 10 cm, carrying a current of 1
   A. What is the magnitude of the
   magnetic field at the centre of the coil?
              [NCERT] (Ans. 6.28 × 10-4 T)
2. A circular loop of one turn carries a
   current of 5.0 A. If the magnetic field at
   the centre is 0.20 mT, find the radius of      7. A straight wire, of length metre, is
   the loop. (Ans. 1.57 cm)                          bent into a circular shape. If the wire
3. What current has to be maintained in a            were to carry a current of 5 A, calculate
   circular coil of wire of 50 turns and 2.54        the magnetic field, due to it, before
   cm in radius in order to just cancel the          bending, at a point distant 0.01 times
   effect of earth's magnetic field at a             the radius of the circle formed from it.
   place where the horizontal component              Also calculate the magnetic field, at the
   of earth's field is 1.86 × 10-5 T ? (Ans.         centre of the circular loop formed, for
   0.015 A)                                          the same value of current.
                   Fig. 4.38
10. A long wire is bent as shown in Fig.
    4.39. Find the magnitude and direction
    of the magnetic field at the centre O of
    the circular part, if a current I is passed     13. A current of I ampere is flowing through
    through the wire.                               the bent wire shown in Fig. 4.42. Find the
                                                    magnitude and direction of the magnetic
                                                    field at point O.
                                                              [Ans.            directed normally
                                                                                      downward]
                                                    14. In Fig. 4.43, the curved portion is a
                                                    semi-circle and the straight wires are long.
                                                    Find the magnetic field at the point O.
[Ans.     (       ) normally into the plane of
                                        paper]      Ans.         ns.
( ) Normally downward]
     find the magnetic field at the point O.             Let the resultant field make angle θ with the
                                                             vertical. Then,
                           ns.
                 (                                   )
                     normally inward
                                                         ∴
                                                         7. Here                  metre                     m
                                                         Magnetic field due to straight wire,
                                                                = 4 ×10-4 T
                                                         Magnetic field at the centre of the circular
                                                         loop,
HINTS
                                                                                                                T
1. As the coil is tightly wound, so radius of
   each turn, r = 10 cm = 0.1 m                          8.       (i) N = 35, I = 1.2 A, a = 40 cm = 0.04
                                                         m,
                                                                r = 40 cm = 0.40 m
                                             T
                                                                          axial               ⁄
     2. Radius,
= 1.57×10-2 m =1.57 cm.
                                                                                                  ⁄
3.                   R
                                                                                          T
                                                         (ii)    centre                       T
closed circuit. The closed curve is called        Application of Ampere's law to a straight
Amperean loop which is a geometrical              conductor. Fig. 4.46 shows a circular loop
entity and not a real wire loop.                  of radius r around an infinitely long straight
                                                  wire carrying current I.         As the field
    Proof for a straight current carrying
conductor. Consider an infinitely long            lines are circular, the field ⃗ at any point of
straight conductor carrying a current I.          the circular loop is directed along the
From Biot-Savart law, the magnitude of the        tangent to the
magnetic field ⃗ due to the current carrying
conductor at a point, distant r from it is
given by
⃗⃗⃗
                                                                    Fig. 4.46
                                                  circle at that point. By symmetry, the
                                                  magnitude of field ⃗ is same at every point
                                                  of the circular loop. Therefore,
                                                        ⃗   ⃗⃗⃗
    Fig. 4.45 Ampere's circuital law.
                                                        From Ampere's circuital law,
As shown in Fig. 4.45, the field B is
directed along the circumference of the           B.2       = μQI
circle of radius r with the wire as centre.
                                                  ∴
The magnitude of the field ⃗ is same for all
points on the circle. To evaluate the line        For Your Knowledge
integral of the magnetic field ⃗ along the
                                                   Ampere's circuital law is not
circle, we consider a small current element
                                                    independent of the Biot- Savart law.
⃗⃗⃗ along the circle. At every point on the
                                                    It can be derived from the
circle, both ⃗ and ⃗⃗⃗ are tangential to the        Biot-Savart law. Its relationship to
circle so that the angle between them is            the Biot-Savart law is similar to the
zero.                                               relationship between Gauss's law
       ⃗⃗⃗                       ⃗⃗⃗                and Coulomb's law.
∴ ⃗                                                Both Ampere's circuital law and
     Hence the line integral of the magnetic        Biot-Savart law relate magnetic field
field along the circular path is                    to the electric current.
                                                   Ampere's and Gauss's laws relate
       ⃗     ⃗⃗⃗                                    one physical quantity (magnetic or
                                                    electric quantity) on the boundary or
                                                    periphery to another physical
                                                    quantity (current or charge), called
                                                    source, in the interior.
                   ∴ ⃗     ⃗⃗⃗
                                                   Ampere's circuital law holds for
    This proves Ampere's law. This law is           steady currents which do not
valid for any assembly of current and for           change with time.
any arbitrary closed loop.                         Although both Ampere's law and
                                                    Biot-Savart law are equivalent in
    9. (b) Calculate, using Ampere's                physical content, yet the Ampere's
circuital theorem, the magnetic field due to        law is more useful under certain
an infinitely long wire carrying a current I.       symmetrical situations. The
where,
l = length of the side ab of the rectangular
loop abed.
    Let number of turns per unit length of
the solenoid = n
Then number of turns in length l of the
solenoid = nl
      Thus the current I of the solenoid                                      Fig. 4.52 A toroidal solenoid.
          end
                                    T
    (c) The outside field near the centre of       Fig. 4.54 A steady current I distributed
a long solenoid is negligible compared to             uniformly across a wire of radius α.
the internal field.
                                                       Solution, (i) Application of Ampere's
Example 32. A coil wrapped around a               law to a long straight cylindrical wire.
toroid has inner radius of 20.0 cm and an         By symmetry, the magnetic lines of force
outer radius of 25.0 cm. If the wire              will be circles, with their centres on the axis
wrapping makes 800 turns and carries a            of the cylinder and in planes perpendicular
current of 12.0 A, what are the maximum           to the axis of the cylinder. So we consider
and minimum values of the magnetic field          Amperean loop as a circle of radius r.
within the toroid?
                                                      Field at outside points. The Amperean
   Solution. Let a and b denote the inner         loop is a circle labelled 2 having radius r >
and outer radii of the toroid. Then               a.
                                                       Length of the loop, L = 2
                                                       Net current enclosed by the loop = I
                                                       By Ampere's circuital law,
                        T      mT                 BL=μo I
                                                  or
                                                  or
                                                                          [For r>α]
                        T        mT               i.e.,                               [For
Example 33. (i) A straight thick long wire of     outside points]
uniform cross- section of radius 'a' is               Field at inside points. The Amperean
carrying a steady current I. Use Ampere's         loop is a circle labelled 1 with r < a.
circuital law to obtain a relation showing
the variation of the magnetic field (B) inside         Length of the loop, L = 2
and outside the wire zvith distance r, (r < a)         Clearly, the current enclosed by loop 1
and (r > a) of the field point from the centre    is less than I. As the current distribution is
of its cross-section. Plot a graph showing        uniform, the fraction of I enclosed is
the variation of field B with distance r.
    (ii) Calculate the ratio of magnetic field
at a point a/2 above the surface of the wire
to that at a point a/2 below its surface.
What is the maximum value of the field of              Applying Ampere's law,
this wire?
          [NCERT ; CBSE D 10 ; OD 16C]
or ( )
outside
                                                                          = 2.86 × 10-5 T.
                                                  Problems For Practice
Fig. 4.55 (a) Sketch of the magnitude of          1. A long solenoid consists of 20 turns per
     the magnetic field for the long                 cm. What current is necessary to
         conductor of radius a.                      produce a magnetic field of 20 mT
                                                     inside the solenoid?      (Ans. 8.0 A)
    (ii) Suppose the point P lies at distance
a 12 above the surface of the wire and            2. A long solenoid is made by closely
point Q lies at distance a / 2 below the             winding a wire of radius 0.5 mm over a
surface. [Fig. 4.55(b)]                              cylindrical non-magnetic frame so that
                                                     successive turns nearly touch each
   Magnetic field at point P at distance r           other. What will be the magnetic field at
=3α/2 from the axis of the wire is                   the centre of the solenoid if a current of
                                                     5 A flows through it?
                                                                                     (Ans. 2 × 10-3 T)
    Magnetic field at point Q at distance r =     3. The magnetic field at the centre of a 50
α / 2 from the axis of the wire is                   cm long solenoid is 4.0 × 10-2 T when a
                                                     current of 8.0 A flows through it. What
                                                     is the number of turns in the solenoid?
                                                     Take = 3.14        (Ans. 1990)
                                                  4. A solenoid is 1.0 m long and 3.0 cm in
                                                     diameter. It has five layers of windings
    Clearly, B is maximum on the surface             of 850 turns each and carries a current
of the wire i.e., at r = α. Hence,                   of 5.0 A. (i) What is B at its centre? (ii)
                                                     What is the magnetic flux Φg for a
                                                     cross-section of the solenoid at the
                                                     centre?
Example 34. A wire of radius 0.5 cm carries
a current of 100 A, which is uniformly                   [Ans. (0 2.67 × 10-2 T, (ii) 1.9 × 10-5 Wb]
distributed over its cross-section. Find the      5. A solenoid is 2.0 m long and 3.0 cm in
magnetic field (i) at 0.1 cm from the axis of        diameter. It has 5 layers of winding of
the wire, (ii) at the surface of the wire and        1000 turns each and carries a current
(iii) at a point outside the wire 0.2 cm from        of 5.0 A. What is the magnetic field at
the surface of the wire.                             the centre? Use the standard value of
Solution. Here R =0.5 cm =0.5 × 10-2 m, I            μ0. [Punjab 97C1
=100 A                                                                              (Ans. 1.57 ×10-2 T)
Current,
upwards.
Example 38. A positive charge of 1.5 μC is
moving with a speed of 2 × 106 ms-1 along
the positive X-axis. A magnetic field,
⃗       ̂      ̂ tesla acts in space. Find
the magnetic force acting on the charge.
     Solution. Here q = 1.5 μC =1.5 × 10-6
C,
                 ̂ms            ⃗                     ̂               ̂ T
                                                                                    Solution. Magnetic field at point P due
Magnetic force on the positive charge is                                                to the current in wire AB,
          ⃗
                                    ̂             ̂                   ̂                                                    T
                        ̂   ̂               ̂             ̂                         This field acts on the proton normally
       ̂         ̂ N                    ̂       ̂ ̂               ̂         ̂   into the plane of paper. According to
                                                                                Fleming's left hand rule, a magnetic force
Example 39. A 5.0 MeV proton is falling                                         acts on the proton towards right in the
vertically downward through a region of                                         plane of paper. The magnitude of this force
magnetic field 1.5 T acting horizontally                                        is
from south to north. Find the magnitude
and the direction of the magnetic force                                         F = qvB sin 90°
exerted on the proton. Take mass of the                                         = 1.6 × 10-19 × 4 × 106 × 4 × 10-6 × 1
proton as 1.6 × 10-27 kg.
                                                                                = 2.56 × 10-18 N.
Solution. Kinetic energy of the proton is
                                                                                Example 41. Copper has 8.0 × 1028
                                                                                electrons per cubic metre. A copper wire of
                       eV                                             J
                                                                                length 1 m and cross-sectional area 8.0
                                                                                        × 10-6 m2 carrying a current and
                                                              J                 lying at right angle to a magnetic field of
                                                                                strength 5 × 10-3 T experiences a force of
or                                                                              8.0 × 10 N. Calculate the drift velocity of
                                                                                free electrons in the wire.
                       ms                                                       Solution, n =8 × 1028m-3, l =1 m
     Force on the proton is                                                     A = 8 × 10-6m2, e = 1.6 × 10-19 C
F = qv B sin 90°                                                                Total charge contained in the wire,
           -19                  7
= 1.6 × 10 × 3.16 ×10 × 1.5 ×                                                   q = Volume of wire × ne= Alne
= 7.58 × 10-12 N                                                                = 8× 10-6 × 1 × 8 × 1028 × 1.6 × 10-19 C
   According to Fleming's left hand rule,                                       = 102.4 × 103 C
the magnetic force on the proton acts
eastwards.                                                                      If vd is the drift speed of electrons, then
Example 40. A long straight wire AB carries                                     F = qvd B sin 90° -qvd B
a current of 4 A. A proton P travels at 4 ×
106 ml s, parallel to the wire, 0.2 m from it                                                                        ms
and in a direction opposite to the current as
shown in Fig. 4.61. Calculate the force                                         = 1.56 × 10-4ms-1.
which the magnetic field of current exerts
                                                                                Problems For Practice
on the proton. Also specify the direction of
the force.                                                                      1. An electron moving with a velocity of
                                                                                   5.0 × 10' ms-1 enters a magnetic field of
                                            [CBSE OD 02]
     1.0 Wb m-2 at an angle of 30°. Calculate      = 1.6 × 10-19 × 5.0 × 107 × 1.0 × sin 30°
     the force on the electron.
                                                   = 4.0 ×10-12 N.
                           (Ans. 4.0 × 10-12N)
                                                   2. (i) Here m = 6.65 × 1027 kg,
                                          -27
2.     n α-particle of mass 6.65 × 10 kg
                                                   q = + 2e = 2×1.6× 10-16C, B = 0.2 T,
     and charge twice that of an electron but
     of positive sign travels at right angles to   v= 6×105 ms-1, 0 = 90°
     a magnetic field with a speed of 6 × 10 5     F = qvB sin 90°
     ms-1. The strength of the magnetic field
     is 0.2 T. (i) Calculate the force on the      = 2 x 1.6 x 10-19 × 6 x 105 × 0.2 × 1N
     α-particle. (ii) Also calculate its           = 3.84 ×10-14 N
     acceleration.
[Ans. (0 3.84 × 10-14 N (ii) 5.77 × 1012 ms-2]                                            ms
3. An electron is moving northwards with
                                                   3. F = qvB sin 90° = 1.6 × 10-19 × 107 × 3 ×
   a velocity of 107 ms-1 in a magnetic field
                                                      1
   of 3 T, directed downwards. Calculate
   the instantaneous force on the                  = 4.8 ×10-12 N
   electron.                                       According to Fleming's left hand rule, the
     (Ans. 4.8 × 10-12 N, vertically upwards)      force acts vertically upwards.
4. A solenoid, of length 1.5 m, has a              4.                            T          T
   radius of 1.5 cm and has a total of 1500
   turns wound on it. It carries a current of      Force, F = evB sin 0° = 0.
   3 A. Calculate the magnitude of the
                                                   5. Magnetic field of the straight wire
   axial magnetic field inside the solenoid.
                                                      carrying a current of 2 A, at a distance
   If an electron were to move with a
                                                      of 10 cm or 0.1 m from it is
   speed of 2 × 104 ms-1 along the axis of
   this current carrying solenoid, what
   would be the force experienced by this                                                   T
   electron ?
                                                   This field acts perpendicular to the
             [CBSE D 08C] (Ans. 0.38 T, 0)         direction of the electron. So magnetic force
5. An electron is moving at 106 ms-1 in a          on the electron is F = qv Bsin 90°
   direction parallel to a current of 5 A,         = 1.6 × 10-19 × 106 × 10-5 × 1 = 1.6 ×10-18 N.
   flowing through an infinitely long
                                                   6. Proceed as in Example 39, on page
   straight wire, separated by a
                                                      4.31.
   perpendicular distance of 10 cm in air.
   Calculate the magnitude of the force            4.13   WORK DONE BY A MAGNETIC
   experienced by the electron.                       FORCE ON A CHARGED PARTICLE
                                                      IS ZERO
           [CBSE D 99] (Ans. 1.6 × 10-18 N)
                                                   14. Show that the work done by a magnetic
6. A proton of energy 3.4 MeV moves
                                                       field on a moving charged particle is
   vertically downwards through a
                                                       always zero.
   horizontal magnetic field of 3 T which
   acts from south to north. What is the               Work done by a magnetic force on a
   force on the proton? Mass of proton is          charged particle. The magnetic force
   1.7 × 10-27 kg; charge on proton is 1.6 ×                  ⃗ always acts perpendicular to
   10-19 C. (Ans. 12.15 × 10-12 N)                 the velocity or the direction of motion of
HINTS                                              charge q. Therefore,
1. q = e = 1.6 × 10-19 C, v - 5.0 × 10 7 ms-1                               ⃗
B = 1.0 Wb m-2, θ = 30°                                 According to Newton's second law,
Force, F = qvB sin θ
Or    *
              ⃗        ⃗
                           +                       Figure 4.62 shows a magnetic field ⃗
                                                   directed normally into the plane of paper,
Or                                                 as shown by small crosses. A charge + q is
                                                   projected with a speed v in the plane of the
Or                                                 paper. The velocity is perpendicular to the
Or K = constant
    Thus a magnetic force does not change
the kinetic energy of the charged particle.
This indicates that the speed of the particle
does not change. According to the
work-energy theorem, the change in kinetic
energy is equal to the work done on the
particle by the net force. Hence the work
done on the charged particle by the
magnetic force is zero.
                                                   Fig. 4.62 A positively charged particle
4.14  MOTION OF A CHARGED                          moving in a magnetic field directed into the
   PARTICLE IN A UNIFORM                           plane of paper.
   MAGNETIC FIELD
                                                   magnetic field. A force F = qvB acts on the
     15. Discuss the motion of a charged           particle perpendicular to both and ⃗ . This
particle in a uniform magnetic field with          force continuously deflects the particle
initial velocity (i) parallel to the field, (ii)   sideways without changing its speed and
perpendicular to the magnetic field and (iii)      the particle will move along a circle
at an arbitrary angle with the field direction.    perpendicular to the field. Thus the
    Motion of a charged particle in a              magnetic force provides the centripetal
uniform magnetic field. When a charged             force. Let r be the radius of the circular
particle having charge q and velocity              path. Now
enters a magnetic field B, it experiences a
                                                      Centripetal force,      = Magnetic force,
force
                                                   qvB
                               ⃗
        The direction of this force is
  perpendicular to both and ⃗ . The                    Thus the radius of the circular orbit is
 magnitude of this force is F = qv B sin θ         inversely proportional to the specific
     Following three cases are possible :          charge (charge to mass ratio q / m) and to
                                                   the magnetic field.
    1. When the initial velocity is parallel to
the magnetic field. Here θ =0°, so F = qvB         Period of revolution =
sin 0° =0.
    Thus the parallel magnetic field does
not exert any force on the moving charged
particle. The charged particle will continue           Clearly, the time period is independent
to move along the line of force.                   of v and r. If the particle moves faster, the
   2. When the initial velocity is                 radius is larger, it has to move along a
perpendicular to the magnetic field. Here θ        larger circle so that the time taken is the
= 90°, so F = qvB sin 90° = qvB = a                same.
maximum force. As the magnetic force acts              The frequency of revolution is
pitch
Only those electrons will pass undeflected          enters a region of uniform magnetic field of
through the slit S2 on which the electric and       10-5 T acting vertically downward as shown
magnetic forces are equal and opposite.             in Fig. 4.65(a). Draw its trajectory and find
The velocity v of the undeflected electrons         out the time it takes to come out of the
is given by                                         region of magnetic field.
                                                                                   [CBSE F 15]
                         or
or
and
                                                                                        s
Pitch    of    helix,
                                                                            s               s
                                                    Example 43. An electron travels in a
5. K.E. gained by an electron when                  circular path of radius 20 cm in a magnetic
   accelerated through a potential                  field 2 × 10-3 T. (i) Calculate the speed of
   difference V,                                    the electron, (ii) What is the potential
                                                    difference through which the electron must
                                  √                 be accelerated to acquire this speed ?
                                                       Solution. Here r =20 cm =20 × 10-2m,
    Units Used                                         B = 2 × 10-3 T, o = 1.6 × 10-19 Q m =
E is in Vm-1 or NC-1, B in tesla, v in ms-1, r in   9.1× 10-31 kg
metre.                                                 (i) Magnetic force on the electron =
Example 42. An electron moving                      Centripetal force on electron
horizontally with a velocity of 4 ×104m/s
                                                                        ̂       ̂        ̂
                          3
           = 13.9 × 10 V -14 kV.                       Thus the force F acts on the charge q
Example 44. An electron after being                along the +ve x-direction.
accelerated through a potential difference             (i) For undeflected proton beam,
of 104 V enters a uniform magnetic field of
0.04 T perpendicular to its direction of           qvB - qE
motion. Calculate the radius of curvature of                         kVm                     Vm
its trajectory.
                                                                      mT                      T
                                  [CBSE F 17]      = 2 × 106 ms-1.
                              4
     Solution. Here V = 10 V, B = 0.04 T,          (ii) Current carried by proton beam,
                 -19                    -31
     e = 1.6 × 10      C m = 9.1 × 10         kg   I =0.8 mA=8 × 10-4 A
   An electron accelerated through a p.d.          Number of protons striking the screen per
V acquires a velocity v given by                     second,
                         or
                                  √                                                              s
                                                               mp = 1.675 × 10-27kg
   As the electron describes a circular
path of radius of r in the perpendicular               Force with which a proton beam strikes
magnetic field B, therefore,                       a target on the screen,
ms
√ √
        √                                                                                           ms
                                                      Velocity component moves the electron
                = 7.5 × 10-4 m = 0.75 mm.         along the field and v∑ along circular path.
                                                  Hence the motion is helical.
9.
                                                  Diameter
        T
10. For the particles to go undeflected,          = 0.36 m = 36 cm.
Force due to electric field = Force due to
magnetic                                                                                                      s
field                                             Pitch
     qE = qvB
                                                  = 0.55 m = 55 cm.
                    Vm
or                   ms
                                   T              4.16 CYCLOTRON
11. For a given p.d., the kinetic energy of a         17. What is a cyclotron? Discuss the
    charged particle is proportional to its       principle, construction, theory and working
    charge.                                       of a cyclotron. What is the maximum kinetic
                                                  energy acquired by the accelerated
∴ Kp:Kd:Ka=e:e:2e = 1:1:2                         charged particles? Give the limitations and
Radius of the circular path of any particle of    uses of a cyclotron.
kinetic energy K,                                     Cyclotron. It is a device used to
frequency. This is the key fact which is          mass. This will throw the particles out of
made use of in the operation of a cyclotron.      resonance with the oscillating field. That is,
                                                  as the ions reach the gap between the
    Working. Suppose a positive ion, say a
                                                  dees, the polarity of the dees is not
proton, enters the gap between the two
                                                  reversed at that instant. Consequently the
dees and finds dee D1 to be negative. It
                                                  ions are not accelerated further.
gets accelerated towards dee D1 As it
enters the dee D2 it does not experience              The above drawback is overcome
any electric field due to shielding effect of     either by increasing magnetic field as in a
the metallic dee. The perpendicular               synchrotron or by decreasing the
magnetic field throws it into a circular path.    frequency of the alternating electric field as
At the instant the proton comes out of dee        in a synchro-cyclotron.
D3 it finds dee D↑ positive and dee D2
                                                      2. Electrons cannot be accelerated in
negative. It now gets accelerated towards
                                                  a cyclotron. A large increase in their energy
dee D2. It moves faster through D?
                                                  increases their velocity to a very large
describing a larger semicircle than before.
                                                  extent. This throws the electrons out of
Thus if the frequency of the applied voltage
                                                  step with the oscillating field.
is kept exactly the same as the frequency
of revolution of the proton, then every time         3. Neutrons, being electrically neutral,
the proton reaches the gap between the            cannot be accelerated in a cyclotron.
two dees, the electric field is reversed and      Uses of cyclotron :
proton receives a push and finally it
acquires very high energy. This is called         1. The high energy particles produced in a
the cyclotron's resonance condition. The             cyclotron are used to bombard nuclei
proton follows a spiral path. The                    and study the resulting nuclear
accelerated proton is ejected through a              reactions and hence investigate
window by a deflecting voltage and hits the          nuclear structure.
target.                                           2. The high energy particles are used to
     Maximum K.E. of the accelerated ions.           produce other high energy particles,
The ions will attain maximum velocity near           such as neutrons, by collisions. These
the periphery of the dees. If V0 is the              fast neutrons are used in atomic
maximum velocity acquired by the ions and            reactors.
r0 is the radius of the dees, then                3. It is used to implant ions into solids and
                                                     modify their properties or even
                       or                            synthesise new materials.
                                                                   neV     or
= 0.66 T.
                                                      Given                              ms
     Kinetic energy of the emerging beam
will be                                           m = 1.67× 10-27 kg
                                                                                revolutions.
                                                  Problems for Practice
                                                  1. An electron of energy 10,000 eV
                   J                  eV
                                                     describes a circular path in a plane at
= 7.4 MeV.                                           right angles to a uniform magnetic field
                                                     of 0.01 T. (a) What is the radius of the
Example 53. In a cyclotron, a magnetic               circular orbit? (b) What is the cyclotron
induction of 1.4 T is used to accelerate             frequency? (c) What is the period of its
protons. How rapidly should the electric             revolution? (d) What is the direction of
field between the dees be reversed? The              revolution as viewed by an observer
mass and charge of proton are 1.67 ×                 looking in the direction of the field?
10-27kg and 1.6 × 10-19 C respectively.
                                                     (Ans. 3.4×10-2m, 2.8×108s-1, 3.6× 10-9s,
    Solution. Here B = 1.4 T, m = 1.67 ×                                  clockwise sense).
10-27 kg,
                                                  2. The protons are accelerated by a
e = 1.6x 10-19 C                                     cyclotron, when a magnetic field of 2.0
    The time required by a charged particle          T is applied perpendicular to the plane
to complete a semicircle in a dee is                 of the dees. Calculate the energy of the
                                                     proton in MeV, if the circular path of the
                                                     protons has a radius of 40 cm before
                                                     the protons leave the cyclotron. Given
                                       s             mass of a proton = 1.67 × 10-27kg.
   Thus the direction of electric field                                         (Ans. 30.6 MeV)
should reverse after every 2.34 × 10-8 s.
                                                  3. A cyclotron has an oscillatory
     The frequency of the applied electric           frequency of 12 MHz and a dee radius
field should be                                      of 50 cm. Calculate the magnetic field
                                                     required to accelerate deutrons of
                                           Hz        mass 3.3 × 10-2 kg and charge 1.6 × 10
                                                     -19
                                                         C. What is the energy of the
Example 54. If the maximum value of                  deutrons emerging from the cyclotron?
accelerating potential provided by a radio
frequency oscillator be 20 kV, find the                                (Ans. 1.56 T, 14.7 MeV)
number of revolutions made by a proton in         4. Alpha particles of mass 6.68 × 10-27 kg
a cyclotron to achieve one fifth of the speed        and charge 3.2 × 10-19 C are
of light. Mass of a proton = 1.67 × 10-27 kg.        accelerated in a cyclotron in which a
    Solution. In a cyclotron, a proton gains         magnetic field of 1.25 T is applied
energy eV, when it crosses a region of               perpendicular to the dees. How rapidly
potential difference V. In one revolution,           should the electric field between the
the particle crosses the gap twice. So the           dees be reversed? What are the
energy gained in each revolution = 2 eV.             velocity and kinetic energy of an alpha
                                                     particle when it moves in a circular orbit
   Suppose      the    particle   makes     n        of radius 25 cm?
     (Ans. 9.5 × 106 Hz, 1.5 × 107 ms-1, 7.5 ×     What is the cause of this force?
                                      10-13 J)
                                                       Force on a current carrying conductor
HINTS                                              in a magnetic field. When a conductor
                                                   carrying a current is placed in an external
      4.                                           magnetic field, it experiences a mechanical
                                                   force. To demonstrate this force, take a
                                 J
                                                   small aluminium rod AB. Suspend it
                                                   horizontally by means of connecting wires
                                              eV   from a stand, as shown in Fig. 4.69.
= 30.6 MeV.
3. As
= 1.56 T.
                                     J
                                                   Fig. 4.69 Force on a current in a
                                         eV        magnetic field.
                    ̂           ̂           ̂   ̂       ̂       ̂
                ⃗       ̂           ̂               ̂       ̂
Magnitude of the magnetic force is
      √                                         √               newton.
Example 56. The horizontal component of                                                    Fig. 4.71
the earth's magnetic field at a certain place                                 Solution. By symmetry, the current
is 3.0 × 10-5 T and the direction of the field                            through each of the four sides will be 1 A.
is from the geographic south to the                                       Also,
geographic north. A very long straight
conductor is carrying a steady current of l                                  l =20 cm =0.20 m, B = 0.25T
A. What is the force per unit length on it                                    Magnitude of force on each side is F =
when it is placed on a horizontal table and                               IIBsin90°
the direction of the current is (a) east to
west, (b) south to north?                                                 = 1 × 0.20 × 0.25×1 =0.05 N
[NCERT]                                                                      By Fleming's left hand rule, forces on
                                                                          ab and dc will be towards left and on ad
    Solution. The force on a conductor of                                 and be downward.
length / placed in a magnetic field B, and
carrying current I, is                                                    Example 58. A magnetic field of 1.0 T is
                                                                          produced by an electromagnet in a
F = IIB sin θ                                                             cylindrical region of radius 4.0 cm, as
The force per unit length will be                                         shown in Fig. 4.72. A wire, carrying current
                                                                          of 2.0 , is placed peiγen- dicular to and
                                                                          intersecting the axis of the cylindrical
                                                                          region. Find the magnetic force acting on
where θ is the angle that the conductor                                   the wire.
makes with the direction of B .                                               Solution. Clearly, the magnetic field
(a) When the current flows east to west, θ                                acts vertically downwards while the current
    =90°.                                                                 flows horizontally, so
∴                                                                            θ =90°.
= 3.0 × 10-5 Nm-1
    According to Fleming's left hand rule,
this force acts vertically downwards.
    (b) When the current flows from south
to north, θ =0°
Example 59. A straight wire of mass 200 g         ∴ F =12 × 0.04 × 0.25 sin 60° = 0.10 N.
and length 1.5 m carries a current of 2 A. It
is suspended in mid-air by a uniform              Example 61. On a smooth plane inclined at
horizontal magnetic field B. What is the          30° with the horizontal, a thin
magnitude of the magnetic field?                  current-carrying metallic rod is placed
       [NCERT; CBSE F 15]                         parallel to the horizontal ground. The plane
                                                  is located in a uniform magnetic field of
    Solution. Suppose that a wire AB              0.15 T in the vertical direction. For what
carries a current of 2 A in the direction as      value of current can the rod remain
shown in Fig. 4.73. The weight mg of the          stationary? The mass per unit length of the
wire acts vertically downwards. Therefore,        rod is 0.03 kg m-1. [NCERT] Solution.
according to Fleming's left hand rule, the        Suppose a rod PQ is placed horizontally on
magnetic field B must act perpendicularly         an inclined plane as shown in Fig. 4.74.
into the plane of paper so that the magnetic      Various forces acting on the current
force F on the wire acts vertically upwards.      carrying rod PQ are
                                                     (i) its weight Mg acting vertically
                                                  downwards; and
                                                    (ii) horizontal force BIl due to the
                                                  magnetic field B .
                                                       In order that the rod remains stationary,
                                                  the component of the weight of the rod
                                                  along the inclined plane must be balanced
                                                  by the component of the force BIl along the
                 Fig. 4.73                        inclined plane, i.e.,
     For mid-air suspension,                      Mg sin θ = BIl cos θ
Magnetic force on the wire = Weight of the
  wire IIB sin 90° = mg
Or
    But m = 200 g =0.2 kg, g=9.8 ms-2, I =
1.5 m,
I =2 A
∴
Example 60. What is the force on a wire of
length 4.0 cm placed inside a solenoid near                         Fig. 4.74
its centre, making an angle of 60° with its       If m is the mass per unit length of the rod,
axis? The wire carries a current of 12 A              then M = ml
and the magnetic field due to the solenoid
has a magnitude of 0.25 T.                             ∴ mlg sin θ = BIl cos θ
    Solution. The force on a conductor of         ∴
length l placed in a magnetic field B, and
carrying current 1, is                            Or
F = IIB sin θ
where θ is the angle that the conductor
makes with the direction of B.                    Example 62 A short conductor of length 5.0
     Since the field due to a solenoid near       cm is placed parallel to a long conductor of
its centre is along its axis, so θ =60°.          length 1.5 m near its centre. The
                                                  conductors carry currents 4.0 A and 3.0 A
Also I=12 A, I= 4.0 cm = 0.04 m, B = 0.25T        respectively in the same direction. What is
                                                       ∴
                                                       11 sin θ 0.1 × 5 × sin 30°
                                                       12.     F = IIB sin θ = 5 × 0.30 × 0.25 sin
                                                       (180° - 120°)
                                                       = 5 × 0.30 × 0.25 × sin 60° = 0.32 N.
                    Fig. 4.78
                                                       4.18 FORCES   BETWEEN   TWO
HINTS                                                     PARALLEL CURRENT-CARRYING
                                                          CONDUCTORS
3. F = IIB sin θ = 12 × 0.04 × 0.25 sin 30°
   = 0.06 N.                                               20. How will you show experimentally
                                                       the existence of (i) attractive forces
4. (a) Magnetic field due to P at Q is
                                                       between parallel currents and (ii) repulsive
                                                       forces between anti-parallel currents?
                                               T
                                                           Forces     between      two     parallel
(b) Force on Q F = IIB sin θ                           current-carrying conductors. It was first
                                                       observed by Ampere in 1820 that two
= 3× 0.05 × 4 × 10-6 × sin 90°
                                                       parallel straight conductors carrying
= 6.0 X10-7T.                                          currents in the same direction attract each
                                                       other and those carrying currents in the
5.                                                 N
                                                       opposite directions repel each other.
                                   W
                                                           Experiment 1. As shown in Fig. 4.79,
6. Force on wire AB, F = IIB sin 90° = IIB             the upper ends of two wires are connected
                                                       to the -ve terminal of a battery and their
Component of the force in the vertically
                                           √
                                                       lower ends are connected to the +ve
upward direction =                                     terminal of the battery through a mercury
                                                       bath. When the circuit is completed, the
If m is the mass per unit length of wire, then         current flows in the two wires in the same
its weight = mlg                                       direction. The two wires are found to be
                               √                       closer to each other, indicating a force of
                                                       attraction between them.
          √             √
or
= 0.2872 kg m-1.
7. In equilibrium,
Magnetic force on wire = Weight of wire
or                   IIB sin 90° = mg
or T
8.    F = IIB sin           . IB sin 90°               Fig. 4.79 Attractive force between parallel
                                                       currents.
                                           N               Experiment 2. As shown in Fig. 4.80,
                                                       two wires are connected to a battery
9.    Proceed as in Problem 8 above.
                                                       through a mercury bath in such a way that
10.    F = IIB sin θ = 5.0 × 0.50 × 0.20 ×             current flows in them in succession. When
sin 90° = 0.50 N.                                      the circuit is closed, the currents in the two
                                                       wires flow in opposite directions. The two
11.     F = IIB sin θ = mg
                                                       wires move away from each other,
                                                       indicating a force of repulsion between
Nm
                            Tm                            Or
Example 65. A current of 5.0 A flows
                                                  = 3.33 × 10-3 m = 3.33 mm.
through each of two parallel long wires.
The wires are 2.5 cm apart. Calculate the         Example 67. A current balance (or ampere
force acting per unit length of each wire.        balance) is a device for measuring
Use the standard value of constant                currents. The current to be measured is
required. What will be the nature of the          arranged to go through two long parallel
force, if both currents flow in the same          wires of equal length in opposite directions
direction?                                        one of which is linked to the pivot of the
                                                  balance. The resulting repulsive force on
                                    [Punjab 99]
                                                  the wire is balanced by putting a suitable
    Solution. Here I1 = I2 = 5 A,                 mass in the scale pan hanging on the other
                                                  side of the pivot. In one measurement, the
r =2.5 cm =2.5 ×10-2m, μ0=4 × 10-7 TmA-1          mass in the scale pan is 30.0 g, the length
Force acting per unit length of each wire,        of the wires is 50.0 cm each, and the
                                                  separation between them is 10.0 mm.
                                                  What is the value of the current being
                                                  measured? Take g =9.80 ms-2 and assume
            Nm                                    that the arms of the balance are equal.
                                                                                            [
    As the currents in both the wires flow in     NCERT]
the same direction, the force will be
attractive.                                          Solution, w = 30.0 g = 0.03 kg, l = 50 cm
                                                  = 0.50 m, r = 10.0 mm = 0.01 m, g = 9.8
Example 66. A long horizontal wire P              ms-2
carries a current of 50 A. It is rigidly fixed.
Another fine wire Q is placed directly above      Force per unit length between two parallel
and parallel to P. The weight of the wire Q       conductors,
is 0.075 Nm-1 and it carries a current of 25
A. Find the position of the wire Q from the
wire P so that Q remains suspended due to
the magnetic repulsion. Also indicate the         ∴ Force on a conductor of length l,
direction of current in Q with respect to P.
[Roorkee 96]
    Solution. The magnetic force per unit                 When the pan is balanced,
length on the wire Q due to the current in
wire P is                                                 Weight in scale pan = Balancing force
                                                  i.e.,
or
                                                  = 29400
                                                  I=√            = 171.46 A.
                                                  Example 68. A rectangular loop of sides 25
                                                  cm and 10 cm carrying a current of 15 A is
                  Fig. 4.82                       placed with its longer side parallel to a long
                                                  straight conductor 2.0 cm apart carrying a
    The currents in P and Q must have             current of 25 A. What is the net force on the
                                                                       Fig. 4.84
                                                      Force per unit length of AB due to current
                                                      in CD is
                    Fig. 4.83
Current through the rectangular loop,             =
15 A
                                                      = 5.0 × 10-4 Nm-1, directed downward Force
Current through the long wire XY,         =25 A       per unit length of AB due to current in EF is
Force on AB,
                length of conductor AB
                                                         = 2.5 × 10-4 Nm-1, directed downward
                                                      Total force per unit length of AB is
                                                                             +
                                                                        f=
                                             -4
=           9.375               ×          10 N
                                                      = 7.5 × 10-4 Nm-1, directed downward Force
                                            (Att
                                                      per unit length of CD due to current in AB is
ractive)
    Force on CD,
                length of conductor CD                    = 5.0 × 10-4 Nm-1, directed upward
                                                      Force per unit length of CD due to current
                                                      in EF is
=           1.5625                  ×       10-4
                                            (Re
pulsive)                                                 = 5.0 × 10-4 Nm-1 directed downward ∴
    ∴ Net force on the loop,                          Total force per unit length of CD = zero.
                                                      Problems For Practice
F=Fl-F2 =9.375 × 10-4 -1.5625 × 10-4
                                                      1. A long horizontal rigidly supported wire
= 7.8125×10-4 N- 7.8 × 10-4 N (Attractive)
                                                         carries a current of 100 A. Directly
    Thus the force on the loop will act                  above it and parallel to it is a fine wire
towards the long conductor (attractive) if               that carries a current of 200 A and
the current in its closer side is in the same            weighs 0.05 Nm-1. How far above the
direction as the current in the long                     wire should the second wire be kept to
conductor, otherwise it will be repulsive.               support it by magnetic repulsion? (Ans.
Example 69. In Fig. 4.84, the wires AB, CD               8 cm)
and EF are long and have identical                    2. A wire AB is carrying a steady current
But NIA = m, the magnetic moment of the                In a uniform magnetic field, the net
   loop, so = mB sin θ                                  magnetic force on a current loop is
                                                        zero but torque acting on it may be
In vector notation, the torque x is given by
                                                        zero or non-zero.
        ⃗⃗   ⃗                                         In a non-uniform magnetic field, the
                                                        net magnetic force on a current is
    The direction of the torque x is such               non-zero but torque acting on it
that it rotates the loop clockwise about the            may be zero or non-zero.
axis of suspension.
                                                      Examples based on
Special Cases
                                                      Torques on current loops
(i) When θ =0°, x =0, i.e., the torque is
    minimum when the plane of the loop is         Formulae Used
    perpendicular to the magnetic field.          Torque on a current loop in a magnetic
(ii) When θ =90°, x = NIB A, i.e., the torque     field, = NIB A sin θ = mB sin θ
     is maximum when the plane of the loop        where m = NIA = magnetic dipole moment
     is parallel to the magnetic field. Thus      of the current loop.
                                                  In vector form,     ⃗⃗    ⃗
For Your Knowledge
                                                  Units Used
    The expression for torque ( =
                                                  Current I is in ampere, area A in m2, field B
     NIBA sin θ) holds for a planar loop
                                                  in tesla, torque x in Nm and magnetic
     of any shape. Thus the torque on a
                                                  moment m in Am2.
     planar current loop depends on
     current, strength of magnetic field          Example 70. The maximum torque acting
     and area of the loop. It is                  on a coil of effective area 0.04 m2 is 4× 10-8
     independent of the shape of the              Nm when the current in it is 100 μ Find
     loop.                                        the magnetic induction in which it is kept.
    For a planar current loop of a given
                                                  Solution. A =0.04 m2,     max   = 4×10-8 Nm,
     perimeter suspended in a magnetic
     field, the torque is maximum when                I = 100 μ =10-4 A, N =1
     the loop is circular in shape. This is           As
     because for a given perimeter, a
     circle has maximum area.                         ∴Magnetic induction,
    The expression          ⃗⃗   ⃗ for the
     torque on a current loop in a
     magnetic field is analogous to the
     expression                 ⃗ for the
     torque on an electric dipole in
     anelectric field. This supports the                                   Wbm
     fact that a current loop is a
     magnetic dipole.                             Example 71. Calculate the torque on a 100
    The torque on a current loop in a            turn rectangular coil of length 40 cm and
     magnetic field is the operating              breadth 20 cm, carrying a current of 10 A,
     principle of the electric motor and          when placed making an angle of 60° with a
     most electric meters used for                magnetic field of 3 T.
     measuring currents and voltages,             Solution. Here I =10 A, N =100, B =3 T,
     called galvanometers.
                                                  A = 40 cm × 20 cm = 800 cm2 = 8 × 10-2 m2
    If the direction of the magnetic field
                                                  θ = 90°-60° =30°
     makes an angle α with the plane of
     the current loop, then                       = Angle between B and the normal to the
     θ + α = 90° or θ = 90° - α                      plane of the coil
          = NIBA sin (90° - α) = NIB A cos
                                                      ∴ Torque,
     α.
    Magnetic field at the centre of the coil,     current of 3 A. A thin coil having 10 turns of
                                                  wire and of radius 0.01 m carries a current
                                                  of 0.4 A. Calculate the torque required to
                                                  hold the coil in the middle of the solenoid
                                                  with its axis perpendicular to the axis of the
                                                  solenoid (μ0 = 4π × 10-7 V-s/ A- m).
                                      T                                           [Roorkee 90]
    The direction of the field is given by           Solution. For solenoid, l = 0.4 m, N1 =
right hand thumb rule.                            500, I1 = 3 A
    (ii) Magnetic moment associated with          For coil, N2 =10, r = 0.01m, I2= 0.4 A
the coil,                                         Field inside the solenoid,
m = NIA = Nl×           = 100 × 3.2 × 3.14 ×
                                                             along the axis of solenoid.
   (0.1)2
= 10 Am2                                              Magnetic moment of coil,
   The direction of m is given by right                                  , along the axis of
hand rule.                                        coil.
(iii) Torque,   = mB sin θ                            As the axis of the coil is perpendicular
                                                  to the axis of solenoid, ⃗⃗ and ⃗ will be
Initially, θ = 0°
                                                  perpendicular to each other.
      Initial torque,
                                                  Required torque,    = m B sin θ
      = mB sin 0° = 0
    Final torque,
      = mB sin 90° = 10 × 2 × 1 = 20 Nm.
    (iv) By Newton's second law,
     orientation will the coil be in stable        4.     = IB A sin θ = 2.0 × 2.0 × 10-2 × 0.40 ×
     equilibrium?                                       0.25 × sin 90°
              [CBSE OD 15C] (Ans. 1 Nm)            = 4.0 ×10-3 Nm.
4. A rectangular coil PQRS is placed in a          5
   uniform magnetic field B, as shown in
   Fig. 4.93. Find the torque on the coil
   when it carries a current of 2.0 A. The
   magnitude of the field B is 2.0 × 10-2 T.       = 0.5T.
                        (Ans. 4.0 × 10-3 Nm)            6.    = NIB (πr2) sin θ
5. A rectangular coil of 100 turns has             = 500 × 1.0 × 0.40 × 3.14 × (0.02)2 × sin 30°
   length 5 cm and width 4 cm. It is placed
                                                   = 0.1256      0.13 Nm.
   with its plane parallel to a uniform
   magnetic field and a current of 2 A is          7. Proceed as in Exercise 4.25 on page
   passed through the coil. If the torque             4.104.
   acting on the coil is 0.2 Nm, find the          4.20      MOVING COIL GALVANOMETER
   magnitude of the magnetic field.
                                                       23. Describe the principle, construction
                                 (Ans. 0.5 T)      and working of a pivoted-type moving coil
6. A circular coil of radius 2.0 cm has 500        galvanometer. Define its figure of merit.
   turns and carries a current of 1.0 A. Its           Moving      coil   galvanometer.       A
   axis makes an angle of 30° with the             galvanometer is a device to detect current
   uniform magnetic field of 0.40 T that           in a circuit. The commonly used moving
   exists in the space. Find the torque            coil galvanometer is named so because it
   acting on the coil. (Ans. 0.13 Nm)              uses a current-carrying coil that rotates (or
7. A circular coil of 200 turns and radius         moves) in a magnetic field due to the
   10 cm is placed in a uniform magnetic           torque acting on it.
   field of 0.5 T, normal to the plane of the          In a D'Arsonval galvanometer, the coil
   coil. If the current in the coil is 3.0 A,      is suspended on a phosphor-bronze wire. It
   calculate the (a) total torque on the coil,     is highly sensitive and requires careful
   (b) total force on the coil, (c) average        handling. In Weston galvanometer, the coil
   force on each electron in the coil, due         is pivoted between two jewellel bearings. It
   to the magnetic field.                          is rugged and portable though less
Assume the area of cross-section of the            sensitive, and is generally used in
wire to be 10-5 m2 and the free electron           laboratories. The basic principle of both
density is 1029/m3.                                types of galvanometers is same.
                             [CBSE OD 08]          Principle. A current carrying coil placed in a
                                        -24        magnetic field experiences a current
     [Ans. (a) zero (b) zero (c) 1.5 ×10      N]   dependent torque, which tends to rotate
HINTS                                              the coil and produces angular deflection.
1. A = 5cm × 12cm = 60×10 4m2                          Construction. As shown in Fig. 4.94, a
                                                   Weston     (pivoted-type)      galvanometer
      = NIB A
                                                   consists of a rectangular coil of fine
= 600 ×10-5 × 0.10×60 ×10-4 = 3.6×10-6Nm.          insulated copper wire wound on a light
                                                   non-magnetic metallic (aluminium) frame.
2.    = NIBA sin θ = 40 × 10 × 0.2 × 100 ×
                                                   The two ends of the axle of this frame are
     10-4 sin 90°
                                                   pivoted between two jewelled bearings.
= 0.8 Nm.                                          The motion of the coil is controlled by a pair
                                                   of hair springs of phosphor-bronze. The
3.    = NIBA sin θ = 200×5×0.2×
                                                   inner ends of the springs are soldered to
     100×10-4sin(90°-60°)
                                                   the two ends of the coil and the outer ends
= 1 Nm.                                            are connected to the binding screws. The
3. Current sensitivity, As
4. Voltage sensitivity,
Units Used
Current I is in ampere, area A in m2, field B
sensitivity change?    [CBSE OD 01]               1. A rectangular coil of area 100 cm2 and
                                                     consisting of 100 turns is suspended in
Solution. Current sensitivity,
                                                     a magnetic field of 5 × 10-2 T. What
                                                     current should be made to pass
                                                     through it in order to keep equilibrium at
Voltage sensitivity,                                 an angle of 45° with the field? Given
                                                     that torsion constant of the suspension
                                                     fibre is 10-8Nm deg-1.
                                                                             (Ans. 9 × 10-6 A)
New current sensitivity,
                                                  2. The coil of a galvanometer consists of
                                                     250 turns of fine wire wound on a 2.0
                                                     cm × 1.0 cm rectangular frame. It is
New voltage sensitivity,                             suspended in a uniform radial magnetic
                                                     field of strength 2,000 G. A current of
                                                     10“4 A produces an angular deflection
                                                     of 30° in the coil. Find the torsional
                                                     constant of its suspension.
   Thus new voltage sensitivity becomes
75% of its initial value i.e., it decreases by                      (Ans. 1.9 ×10-6 Nm rad-1)
25%.                                              3. A moving coil galvanometer is placed in
Example 86. The coil of a galvanometer is            a radial magnetic field of 0.2 T. The
0.02 m× 0.08 m It consists of200 turns of            galvanometer coil has 200 turns and
fine wire and is in a magnetic field of 0.2          area of 1.6×10-4m2. The torsion
tesla. The restoring torque constant of the          constant of the suspension fibre is 10-6
suspension fibre is 10-6 Nm deg-1.                   Nm deg-3 Determine the maximum
Assuming the magnetic field to be radial,            current that can be measured by this
(a) what is the maximum current that can             galvanometer if its scale can
be measured by this galvanometer, if the             accommodate a deflection of 45°. (Ans.
scale can accommodate 30° deflection?                7 × 10-J A)
(b) What is the smallest current that can be      4. The coil of moving coil galvanometer is
detected, if the minimum observable                  40 mm long and 25 mm wide. It has
deflection is 0.1 degree?                            100 turns and is suspended in a radial
                            [CBSE OD 13C]            magnetic field of 10-2 T. If the
                                                     suspension fibre has a torsional
   Solution. Here A =0.02 × 0.08 m2 =1.6             constant of 10-8 Nm deg-1, find the
× 10-3m2                                             current sensitivity of the moving coil
   N = 200, B = 0.2 T, k = 10-6 Nm deg-1             galvanometer. ( ns. 0.1 deg μ _1)
   (a) The maximum current (Imax) that can        5. A coil of a moving coil galvanometer
be measured is given by                              twists through 45° when a current of 1
                                                     micro-ampere is passed through it. If
                                                     the area of the coil is 10-5m2 and it has
                                                     1000 turns, find the magnetic field of
                                                     the magnet of the galvanometer. The
                                                     restoring torque per unit twist of the
              = 4.69 × 10-4 A.                       galvanometer coil is 10-4Nm deg-1.
   (b) The smallest current (Imin) that can            (Ans. 45 T)
be detected is given by                           6. The coil of a pivoted-type galvanometer
                                                     has 50 turns and encloses an area of 6
                                                     m2. The magnetic field in the region in
                                                     which the coil swings is 0.01 T and is
                                                     radial. The torsional constant of the hair
                                                     spring is 1.0×10-8Nm deg-1. Find the
Problems for Practice
   <<G.
 Because of its very small
   resistance, an ammeter placed in a
   series circuit does not practically
   change the current in the circuit to                                  Fig. 4.97
   be measured.                                       A galvanometer can be converted into
                                                  a voltmeter by connecting a high
 The resistance of an ideal ammeter              resistance in series with it. The value of this
   is zero.                                       resistance is so adjusted that only current /
                                                  which produces full scale deflection in the
 Higher the range of ammeter to be               galvanometer,      passes      through      the
   prepared from a given                          galvanometer.
   galvanometer, lower is the value of                 Let
   the shunt resistance required for the          G = resistance of the galvanometer
   purpose.                                       lg = the current with which galvanometer
                                                  gives full scale deflection
 The ammeter of lower range has a
                                                     0 - V = required range of the voltmeter,
   higher resistance than the ammeter             and
   of higher range.                               R = the high series resistance which
 The range of an ammeter can be                    restricts the current to safe limit I.
Here
2. Resistance of an ammeter, or
                                                [IIT]
   Solution. As the current sensitivity is
10 div per mA and there are 100 divisions
on the scale, so current required for full            (ii) The combined resistance RA of the
scale deflection is                                 ammeter and the shunt is given by
        × 100 mA = 10 mA = 10× 10-3A
=0.01 A
    As voltage sensitivity is 2 div per mV,         or           RA =4/25 = 0.16 Ω.
so voltage required for full scale deflection       Example 90. In the circuit (Fig. 4.98), the
is                                                  current is to be measured. What is the
                                                    value of the current if the ammeter shown
       × 100 mV = 50 mV = 50 × 10-3V                (a) is a galvanometer with a resistance R =
     Galvanometer resistance,                       60.00 Ω;
all, on the scale of galvanometer, what           to produce full scale deflection is given by
maximum current can this galvanometer
read?
                                [CBSE D 01C]
    Solution. As the galvanometer has 50          ∴                  …
divisions, current required to produce full
scale deflection is                               When a resistance R is connected in
                                                  series,
        x 50 mA = 5 mA =0.005 A
                                                                       ...(2)
              and           s
                                                  From equations (1) and (2),
    Let I be the maximum current that the
galvanometer can read.
                                                  Rg + 5 “ Rg + 1050
                                                  or 1.25 R + 1312.5 = 25 R + 125
or                                   m
                                                  or 23.75 Rg = 1187.5
Example 92. A galvanometer having a
resistance of 50 Ω gives a full deflection for    or
a current of 0.05 A Calculate the length of
                                                  Example 94. When a galvanometer having
the shunt wire of 2 mm diameter required
                                                  30 divisions scale and 100 Ω resistance is
to convert the galvanometer to an ammeter
                                                  connected in series to a battery of emf 3 V
reading current upto 5 A Specific
                                                  through a resistance of 200 Ω, shows full
resistance for the material of the wire is 5 ×
                                                  scale deflection. Find the figure of merit of
10 7Ω m
                                                  the galvanometer in μ
                                    [Punjab 96]
                                                  Solution. Here n = 30, Rg =100Ω,
Solution.      = 50 Ω, I = 0.05 A, I = 5 A                              S=3V,
                                                  R =200 Ω, k =?
                                                     The current required to produce full
                                                  scale deflection in the galvanometer is
Now                                      m
      mm            m
As          , so length of required shunt is
                                                  As Ig = nk, therefore, the figure of merit is
                                m
                                                               divisions
                        m       m                 Example 95. The deflection produced in a
                                                  galvanometer is reduced to 45 divisions
Example 93. A moving coil galvanometer            from 55 when a shunt of 8 Ω is used.
when shunted with a resistance of 5 Ω             Calculate  the      resistance  of   the
gives a full scale deflection for 250 mA and      galvanometer.
when a resistance of 1050 Ω is connected          Solution. Without shunt,      = I = 55k
in series, it gives a full scale deflection for
25 volt. Find the resistance of the               where k is the figure of merit of the
galvanometer and the current required to             galvanometer. With shunt,
produce afldl scale deflection when it is
used alone.
     Solution. With shunt, current required
                                                  or          Rg = 40 Ω.
                                                  Example 98. A voltmeter reads 5.0 V at full
When shunt of 8 Ω is used,                        scale deflection and is graded according to
                                                  its resistance per volt at full scale deflection
                                                  as 5000 Ω V-1. How will you convert it into a
                                                  voltmeter that reads 20 V at full scale
or                      or 88 = 2 R + 16          deflection? Will it still be graded as 5000
                                                  ΩV? Will you prefer this voltmeter to one
                                                  that is graded as 2000 ΩV-1?
or
                                                                       [NCERT ; CBSE D 01C]
Example 96. A galvanometer of resistance
'G' can be converted into a voltmeter of              Solution. Resistance per volt is another
range (0-V) volts by connecting a                 way of specifying the current at full scale
resistance 'R' in series with it. How much        deflection. The grading of 5000 ΩV~1 at full
resistance will be required to change its         scale deflection means that the current
range from 0 to V/2?                              required for full scale deflection is
                                   [CBSEOD15C]                                   m
Solution. In first case,                              In order to convert it into a voltmeter of
                                                  range 0 to 20 V, a resistance R has to be
                                                  connected in series with it. Then on
                                                  applying an extra P.D. of 15 V (20 V - 5 V),
   Let R’ be the required resistance to           the current through it is again 0.2 mA at full
change the range from 0 to V12. So in             scale deflection.
second case,
                                                  R × 0.2 × 10-3 = 15
                           ⁄
                                                  or
                               ⁄                       Thus (i) to convert the given voltmeter
                                                  (0 - 5 V range) into a voltmeter of range 0 to
                                                  20 V, a resistance of 75,000 Ω should be
                                                  connected in series with the given meter.
               Hence,                                  (ii) Original resistance of voltmeter
Example 97. A galvanometer can be                     = 5000 ΩV-1 × 5 V =25,000 Ω ∴ Total
converted into a voltmeter of certain range       resistance after conversion
by connecting a resistance of 980 Ω in
                                                       = 25,000 + 75,000 = 100,000 Ω
series with it. When the resistance is 470 Ω
connected in series, the range is halved.              Resistance per volt of new meter
Find the resistance of the galvanometer.
                                                                                     V
    Solution. The current for full scale
deflection of a voltmeter is given by             i.e., it has the same grading as before.
                                                      (iii) The higher the 'resistance per volt'
                                                  of the meter, the lesser is the current it
                                                  draws from the circuit and the better it is.
     In first case,
                                                  So this meter is more accurate than the
                                   ⁄              one graded as 2000 ΩV-1.
     In second case,
                                                  Example 99. A galvanometer having 30
                                                  divisions has a current sensitivity of 20 μ J
or
                                                  division. It has a resistance of
2 R + 940 = R + 980                               20 Ω. How will you convert it into an
                                                  ∴
                                                  or           4× 10-4X + 4 = 120
                                                  or                                       k
                                                                 m
                                                                m
                                                               m
                 Fig. 4.100                       Example 104. (a) A battery of emf 9 V and
                                                  negligible internal resistance is connected
Solution. Here Ig =lmA =10-3 , G = 10Ω            to a 3 k Ω resistor. The potential drop
                                                  across a part of the resistor (between
But
                                                  points A and B in Fig. 4.101) is measured
               V                                  by (i) a 20 kΩ voltmeter; (ii) a 1 kΩ
                                                  voltmeter. In (iii) both the voltmeters are
                                                  connected across AB. In which case would
                                                  you get the (1) highest, (2) lowest reading?
                           V
                                       k
                               V
                                   k
Example 103. A multirange currentmeter            Fig. 4.101
can be constructed by using a
                                                      (b) Do your answers to this problem
galvanometer circuit as shown in the
                                                  alter if the potential drop across the entire
figure. We want a currentmeter that can
                                                  resistor is measured? What if the battery
measure 10 mA, 100 mA and 1A using a
                                                  has non-negligible resistance?
galvanometer of resistance 10Ω and that
produces maximum deflection for current                                              [NCERT]
of l mA. Find S1, S2 and S3 that have to be
                                                      Solution, (a) The voltmeter, which has
used. [Example Problem]
                                                  maximum resistance, will draw minimum
                                                  current and allow maximum current to flow
                                                  through resistor AB. Consequently, there
                                                  will be maximum potential difference
                                                  across AB.
      In case (i), resistance of voltmeter,       the circuit (b) is suitable only for the
                                                  measurement of the small resistance X.
Rv =20 kΩ
                                                     We can justify the above arguments
      In case (ii), Ry = 1 k Ω
                                                  quantitatively as follows :
      In case (iii),                   or             (i) Measurement of X. Let X = 5 Ω. In
  k                                               circuit (a), the ammeter reading is I, and
                                                  the voltmeter reading is I, (X + 0.5).
(1) As the resistance of voltmeter is                  Voltmeter reading
    maximum in case (i), it will show
    maximum reading.                                    mmeter reading
mmeter reading
    area of cross-section 0.2 mm2 required               reading on the voltmeter? (Ans. 11.52
    to make a shunt for this purpose.                    V)
                                                    HINTS
                                 (Ans. 1.25 m)
13. A galvanometer gives a full scale               1.
    deflection with a current of 1 A. It is
    converted into ammeter of range 10 A.
                                                    2. As           or
    Find the ratio of the resistance of the
    ammeter to the resistance of the shunt
                                                    3    (i) For conversion into ammeter :
    used. (Ans. 9:10)
                                                                         m
14. A galvanometer has a resistance of 8
    Ω. It gives a full scale deflection for a
    current of 10 mA. It is to be converted
    into an ammeter of range 5 A. The only
    shunt resistance available is of 0.02 Ω,
    which is not suitable for this
    conversion. Find the value of                   = 0.1Ω.
    resistance R that must be connected in          (ii) For conversion into voltmeter :
    series with the galvanometer (Fig.
    4.104) to get ammeter of desired                                                         V
    range.      ( ns. 1.98 Ω)
15. The circuit shown in Fig. 4.105 is used
    to measure the resistance R. The
    ammeter reads 0.13 A and the                    4. For conversion into voltmeter :
    voltmeter reads 117 V. The resistance
    of the ammeter is 0.015 Ω and that of
    the voltmeter is 9000 Ω. Find the value
    of R.       ( ns. 1000 Ω)                       For conversion into ammeter :
16. The scale of a galvanometer is divided
    into 150 equal divisions. The
    galvanometer        has    the     current
                                                                   mV
    sensitivity of 10 divisions per mA and          5.                       m
    the voltage sensitivity of 2 divisions per
    mV. How the galvanometer can be
    designed to read (i) 6 A division-1 and
    (ii) 1 V per division-1?    [Roorkee 99]
[Ans. (i) 8.33 × 10- 5 Ω in parallel (ii) 9995 Ω    6.               m
                                       in series]   ∴ Maximum current,
17. Two resistance coils of 100 Ω and 200
    Ω respectively are connected in series                                                       m
    across 100 V. A moving coil voltmeter
    of 200 Ω is connected in turn across            7. To convert an ammeter of lower current
    each coil. What will it read in each               range to higher current range, a shunt
    case?                                              has to be connected across it. The
                             (Ans. 25 V, 50 V)         shunt resistance is
Or 16. m
Current sensitivity mV V
                                       div
           div
10. Here I ∞ 60 and I                  10
                                                               (i) Required current range, 7 = 6 × 150 =
                          and                                 900 A
     S                    or       s
                                                              (ii) Required voltage range, V = 1 × 150 =
11.                                          or 2% of the     150 V.
      total current passes                    through   the
      galvanometer.
                           ( )                                17. Proceed as in Example 100 on page
12.      S                                                    4.68.
                 ⁄
                                                              Reading of the voltmeter
                  ⁄
                                                              = IR =0.4 x28.8 = 11.52 V.
14. P.D. across the series combination
of G and R = P.D. across the shunt S                          VERY SHORT          ANSWER    CONCEPTUAL
                                                              PROBLEMS
Ig(Rg + R) = (I- Ig) Rs
                                                                 Problem 1. In what respect does a
0.01 (8 + R) = (5 - 0.01) × 0.02 = 4.99 ×                     wire carrying a current differ from a
0.02                                                          wire, which carries no current?
                                                                                               [Punjab 96]
                                                                  Solution. A current carrying wire
or R = 9.98-8 = 1.98 Ω.                                       produces a magnetic field around it. On the
15.      Current through the voltmeter                        other hand, no magnetic field is associated
                                                              with a wire that carries no current.
             V
                                                                    Problem 2. Is the source of magnetic
    Solution, (i) The magnetic field is zero          Solution.   Lorentz    magnetic    force,
inside the pipe. (ii) A finite magnetic field                ⃗
exists outside the pipe.
                                                      As the magnetic force       acts in a
    Problem 10. What is the basic                 direction perpendicular to the direction of
difference between magnetic field and             the velocity or the direction of motion of
electric field ? [Punjab 02]                      the charged particle, so work done is zero.
     Solution. Whether a charged particle is      W=       ⃗⃗⃗⃗ = Fds cos 90°= 0
at rest or in motion, an electric field always
exerts a force on it and changes its speed            Problem 15. A charged particle in a
and hence its kinetic energy. A magnetic          plasma trapped in a 'magnetic bottle'
field exerts a force on a charged particle        leaks out after a millisecond. What is
only when it is in motion. In a magnetic          the total work done by the magnetic
field, there is no change in the speed or the     field during the time the particle is
kinetic energy of the charged particle.           trapped?
   Problem 11. The force F experienced                Solution. Work done is zero. Since a
by a particle of charge q moving with             magnetic field exerts force perpendicular to
                                                  the direction of motion of the charged
velocity in magnetic field ⃗ is given
                                                  particle, no work is done by it on the
by           ⃗ . Which pairs of vectors
                                                  charged particle.
are always at right angles to each
other? [CBSE D 93C]                                   Problem 16. A particle of mass 'm'
                                                  and charge 'q' moving with velocity ‘v’
    Solution. The vector is perpendicular         enters the region of uniform magnetic
to the plane of and ⃗ .       Hence  the          field at right angle to the direction of its
                            ⃗
pairs and ; and and are always at                 motion. How does its kinetic energy get
right angles to each other.                       affected? [CBSE D 15C]
   Problem 12. In a chamber of uniform                Solution. K.E. of the particle is not
magnetic field ⃗ , an electron beam               affected because no work is done by the
enters with velocity       . Write the            perpendicular magnetic force acting on it.
expression for the force experienced by               Problem 17. An electron moving with
the electron.                                     a velocity of 107 ms-1 enters a uniform
              [CBSE D 93C ; Haryana 02]           magnetic field of 1T, along a direction
                                                  parallel to the field. What would be its
   Solution.              ⃗ , because the         trajectory in this field?
charge on an electron is - e.
                                                                   [CBSE Sample Paper 98]
   Problem 13. No force is exerted by a
                                                                                 √
                                                                       √
of the solenoid flows in the same direction.          Solution. A galvanometer in which coil
So different turns attract one another and        comes to rest at once, without suffering
the solenoid contracts.                           oscillations, on passing current through it is
                                                  called a dead beat galvanometer.
   Problem 43. Does the torque on a
planar current loop in a magnetic field                Problem 49. A neutron, an electron and
change, when its shape is changed                 an alpha particle moving with equal
without changing its area?                        velocities, enter a uniform magnetic field
                                                  going into the plane of the paper as shown
    Solution. No. The torque on a planar
                                                  in Fig. 4.109(a). Trace their paths in the
current loop remains same as long as its
                                                  field and justify your answer.
area remains unchanged.
                                                                                  [CBSE D 16]
   Problem 44. Why do we prefer
phosphor-bronze  alloy for  the
suspension wire of a moving coil
galvanometer?
    Solution. The phosphor bronze alloy is
used for the suspension wire of a moving
coil galvanometer because of its following
properties :
(i) Small restoring torque per unit twist.            Solution. Neutrons, being neutral, are
    This makes galvanometer highly                not deflected by the magnetic field. Alpha
    sensitive.                                    particles and electrons will tend to move
                                                  along circular paths in opposite directions
(ii) High tensile strength so that even thin      as shown in Fig. 4.109(b), in accordance
     fibre does not break under the weight of     with Fleming's left hand rule.
     the suspended coil.
                                                      Problem 50. Why is the coil wrapped on
(iii) Rust resisting.                             a conducting frame in a galvanometer?
   Problem 45. What is the main                       Solution. Eddy currents set up in the
function of soft iron cylinder between            conducting frame help in bringing the coil
the poles of a galvanometer?                      to rest at once i.e., help in making the
    Solution. Due to the high permeability        galvanometer dead beat.
of soft iron, the magnetic lines of force              Problem 51 Why the earth's magnetic
crowd through the soft iron core. This            field does not affect the working of a
increases the magnetic field and hence            moving coil galvanometer?
sensitivity of the galvanometer.
                                                      Solution. This is because the earth's
    Problem 46. Why are pole pieces of            magnetic field is much weaker than the
galvanometer made concave? [Haryana               strong radial magnetic field used in the
93, 94]                                           galvanometer.
    Solution. Concave poles produce                   Problem 52. Two wires of equal lengths
strong, uniform and radial magnetic field.        are bent in the form of two loops. One of
    Problem 47. What is the importance            the loops is square shaped whereas the
of radial magnetic field in a moving coil         other loop is circular. These are suspended
galvanometer?                                     in a uniform magnetic field and the same
                                                  current is passed through them. Which
    Solution. Radial magnetic field makes         loop will experience greater torque? Give
the arm of the couple fixed and hence the         reasons.
torque on the coil is always same in all
positions of the coil in the magnetic field.                                    [CBSE OD 05]
This provides a linear current scale.                Solution. For a wire of given length,
    Problem 48. What is a dead beat               area of the circular loop > area of the
galvanometer?                                     square loop
    Problem 62. What happens when a               resistance is smaller than that of the
voltmeter is connected in series in a             galvanometer. The voltmeter is obtained
circuit?                                          by connecting a high resistance in series
                                                  with the galvanometer, so its resistance is
     Solution. A voltmeter is a high
                                                  higher than that of the galvanometer.
resistance device, when it is connected in
series with a circuit, the resistance of the          Problem 67. A galvanometer is first
circuit becomes very high and the current         converted into a voltmeter of range 0 - 3 V
in the circuit reduces to a very small value.     and then into a voltmeter of range 0 - 6 V.
So voltmeter will not measure the actual          In which case the resistance would be
value of potential difference. Moreover, the      higher one?
voltmeter connected this way will not read
                                                      Solution. The voltmeter with range 0 - 6
the p.d. that we wish to measure across
                                                  V will have a higher resistance. This is
any particular component.
                                                  because a higher range voltmeter requires
    Problem 63. A galvanometer gives full         a higher resistance to be put in series with
scale deflection with the current . Can it        the galvanometer.
be converted into an ammeter of range I <              Problem 68. An ammeter and a
  ?                                               milliammeter are converted from the same
Solution. The required value of shunt is          galvanometer. Out of the two, which
                                                  current measuring instrument has a higher
   given by g
                                                  resistance?
   For I < , the above formula gives a                                         [CBSE OD 02]
negative value for Rs.                                Solution. Milliammeter has higher
  Hence it cannot be converted into an            resistance because an ammeter requires a
ammeter of range I < Ig.                          shunt of smaller resistance.
    Problem      64.           Can         we          Problem 69. A voltmeter and a
increase/decrease the        range    of   an     millivoltmeter are converted from the same
ammeter?                                          galvanometer. Out of the two, which
                                                  potential difference measuring device has
    Solution. We can only increase the            a higher resistance?
range of ammeter by connecting a suitable
resistance in parallel with the ammeter. We           Solution. Voltmeter has a higher
cannot decrease the range of the ammeter          resistance because it requires a larger
because for I <        , the value of shunt       resistance to be put in series with the
resistance becomes negative.                      galvanometer.
less than the actual current. It has a small              ammeter or a milliammeter, has a higher
resistance. When it is connected in a                     resistance and why?
circuit, it decreases the current slightly.
                                                                                        [CBSE OD 06]
    Problem 72. When a voltmeter is put
across a part of the circuit, does it read                   Solution. Shunt resistance,     s
slightly less or more than the actual voltage
drop across that part? Give reason.                           Clearly, the shunt needed to convert
                                                          galvanometer into a milliammeter has a
    Solution. Voltmeter reads slightly less               larger value than that required to convert
than the actual voltage drop. When it is                  into an ammeter. As the shunt resistance is
connected across a part of the circuit, it                connected      in   parallel   with    the
draws some current from that part. As a                   galvanometer, so the milliammeter will
result, potential drop across that part                   have a higher resistance than the
slightly decreases.                                       ammeter.
   Problem 73. Which one of the two, an
                    LT I J                                                  or
     Problem 2. A wire of length L is bent
round in the form of a coil having N turns of
same radius. If a steady current I flows
through it in a clockwise direction, find the             When the wire is bent in form of two-turn
magnitude and direction of the magnetic                     coil,
field produced at its centre. [CBSE F 09]
   Solution. If r is the radius of the coil,                                     or
then
                                                                    ∴
Fig. 4.113
                                                            ⃗       ⃗
                                                                                         ⁄
                                                                                                 √
                                                   The net magnetic field at O is
                       Fig. 4.110                       ⃗       ⃗        ⃗
Solution.                                                                            √       √
                                                       (Pointing towards P) Problem 6. Two
                                                   small identical circular loops, marked (1)
                                                   and (2), carrying equal currents, are placed
                                                   with the geometrical axes perpendicular to
                                                   each other as shown in Fig. 4.114. Find the
                                                   magnitude and direction of the net
                                                   magnetic field produced at the point O.
                       Fig. 4.111                  [CBSE D 05, 08]
 ⃗                 ⁄           (Pointing towards
                           √
P)
 ⃗                 ⁄                (Pointing
                           √
towards Q)
     Net field at O is
 ⃗      ⃗      ⃗                    (Pointing
                           √
towards Q)                                                              Fig. 4.114
    Problem 5. Two identical circular loops,          Solution. The fields produced by the
P and Q, each of radius r and carrying             two coils are shown in Fig. 4.115.
currents are kept in the parallel planes
having a common axis passing through
                       Fig. 4.112
O. The direction of current in P is
                                                                        Fig. 4.115
                                       √
              √                                ⁄      Problem 9. Describe qualitatively the
                                                   path of a charged particle moving in :
                             √
   For                                             (a) a uniform electrostatic field, with initial
    Problem 7. Figure 4.116 shows a cube               velocity
made from a uniform wire. Find the                 (i) parallel to the field,
magnetic field at the centre of the cube, if a
battery is connected between the points A          (ii) perpendicular to the field,
and G.                                             (iii) at an arbitrary angle with the field
                                                         direction.
                                                   (b) a uniform magnetic field, with initial
                                                       velocity
                                                   (i) parallel to (or along) the field, [CBSE
                                                       OD 95]
                                                   (ii) perpendicular to the field,
                                                                                      [CBSE D 93]
                                                   (iii) at an arbitrary angle with the field.
                         Fig. 4.116                      [CBSE F 93]
     Solution. Consider the conducting pairs       (c) a region with uniform electrostatic and
AB and HG, AE and CG, AD and EG, BF                    magnetic fields parallel to each other,
and DH, EF and CD, EH and BC. The                      with initial velocity
currents in the two conductors of each pair
produce equal and opposite magnetic                (i) parallel,
fields at the centre. Hence the resultant          (ii) perpendicular,
magnetic field at the centrh will be zero.
                                                   (iii) at an arbitrary angle with the common
     Problem 8. An electron revolves in a                direction of the fields.
circular orbit of radius r with angular speed
cα Using the expression for the magnetic           Solution, (a) (i) The particle will move along
                                                      the line of force of the electrostatic field
field due to a circular current loop, deduce
                                                      and hence its path will be a straight
an expression for the magnetic field at the
                                                      line.
centre of the electron orbit.
                                                   (ii) When the initial velocity is
    Solution. Magnetic field at the centre of
a circular current loop is                              perpendicular to the electrostatic field,
                                                        the charged particle will follow a
                                                        parabolic path.
                                                   (iii) Again the path of charged particle will
Period of revolution of the electron,                    be parabolic. The initial velocity of the
                                                         charged particle has two components.
                                                         The component parallel to the field will
                                                         make it move along the field while the
    The motion of the electron in a circular
                                                         perpendicular component will make it
orbit is equivalent to a current loop carrying
                                                         move along a parabolic path.
current,
                                                   (b) (i) Parallel magnetic field does not exert
∴
     Problem 13. (a) A point charge q
moving with speed v v enters a uniform
magnetic field B that is acting into the plane
of the paper as shown. What is the path
followed by the charge q and in which
plane does it move?
   (b) How does the path followed by the                             Fig. 4.118
charge get affected if its velocity has a               Solution.   For     undeflected   charged
component parallel to ⃗ ?                           particle,
                                                                       or
                                                                       ∴
                                                        As the electric field is switched off, the
                                                    charged particle is moving along x-axis.
                                                    The perpendicular magnetic field will
                  Fig. 4.117                        deflect it along a circular trajectory in the
                                                    XY plane.
    (c) If an electric ⃗ is also applied such
that the particle continues moving along                Problem 15. A proton and a deuteron
the original straight line path, what would         having equal momenta enter in a region of
be the magnitude and direction of the               uniform magnetic field at right angle to the
                                                    direction of the field. Depict their
electric field ⃗ ?       [CBSE F l6]
                                                    trajectories in the field.
    Solution, (a) The charge q moves in the
                                                                                    [CBSE D 13]
XY plane describing a circular path in the
anticlockwise sense.                                    Solution.
     (b) The path of the charged particle
will become helical.
    (c) By Fleming's left hand rule, the                For same p and B,
magnetic Lorentz force acts on charge q in
-Y direction.
   ∴ Applied electric field should be in the
+ Y direction to make the charged particle
move along the original straight line path.
                   F =F
                               or
    Problem 14. A particle of charge q is                            Fig. 4.119
moving with velocity v in the presence of
                                                        Hence both particles will move along
crossed electric field E and magnetic field
                                                    circular trajectories of same radius as
B as shown. Write the condition under
                                                    shown in Fig. 4.119.
which the particle will continue moving
along x-axis. How would the trajectory of                 roblem 16. n α-particle and a proton
the particle be affected if the electric field is   moving with the same speed enter the
switched off?                                       same magnetic field region at right angles
                                                    to the direction of the field, as shown in Fig.
                                    [CBSE SP 18]
                                                    4.120(a). Show the trajectories followed by
                                                    the two particles in the region of the
                                                    magnetic field. Find the ratio of the radii of
                                                    the circular paths which the two particles
Or                         or
                                                                     Fig. 4.121
For same momentum p and field B,
                                                        Solution. When the current flows
                                                    anticlockwise, force on side PQ = IIB,
                                                    acting upwards.
                                                        When the current flows clockwise,
(ii) Kinetic energy,                   or
                                                    force on side PQ = IIB, acting downwards.
      √                                             ∴ Change in the tension of the spring = IIB
                                                       + IIB = 2 IIB.
                                   √
∴Radius,                   √                            Problem 20. Explain, why two wires
                                                    carrying currents in opposite directions
      For same kinetic energy K and field B,        repel each other.
                                                                   Fig. 4.124
                                                      Solution. The windings of a helix carry
                                                  currents in the same direction, they exert
                                                  attractive forces pulling the lower end out
                                                  of mercury. Consequently, the circuit
                                                  breaks and the current and hence the force
                                                  of attraction disappears. The helix regains
                                                  its original condition with its lower end
                                                  dipping in mercury. The process repeats
                 Fig. 4.122                       and oscillatory motion is set up.
    Problem 21. Two parallel conducting               Problem 23. Suppose a helical spring is
wires carrying currents in same direction         suspended from the roof of a room and
attract each other. Why?                          very small weight is attached to its lower
                                                  end. What will happen to the spring when a
    Solution. Each current carrying               current is passed through it? Give reason
conductor produces a magnetic field               to support your answer.
around it. So each conductor experiences
a force due to the magnetic field of the                          [CBSE Sample Paper 1998]
other. According to Fleming's left hand                Solution. When a current is passed
rule, the direction of the force is               through the spring, it will flow in the
perpendicular to the two wires and is             adjacent turns of the spring in the same
inwards. Hence the two wires attract each         direction. So there will be magnetic
other, as shown in Fig. 4.123.                    attraction between adjacent turns. The
                                                  spring will contract and the weight will be
                                                  lifted up.
                                                      Problem 24. A horizontal wire AB of
                                                  length T' and mass 'm' carries a steady
                                                  current Iv free to move in vertical plane is in
                                                  equilibrium at a height of 'h' over another
                                                  parallel long wire CD carrying a steady
                                                  current I2, which is fixed in a horizontal
                                                  plane as shown. Derive the expression for
                 Fig. 4.123                       the force acting per unit length on the wire
                                                  AB and write the condition for which wire
    Problem 22. A loosely wound helix             AB is in equilibrium. [CBSE SP 18]
made of stiff wire is mounted vertically with
its lower end just touching a dish of
mercury, as shown in Fig. 4.124. When a
current from a battery is started in the coil
through the mercury, the wire executes
oscillatory motion with the lower end                              Fig. 4.125
jumping out of and into the mercury. Give
reason.                                               Solution. Magnetic field produced by
                                                  current I2 at any point on the wire AB is
                                                           , normally into the plane of paper
                                                      Force exerted by field B2 on length / of
                                                  wire AB,
                                                  towards conductor 1.
     Problem 25. Figure 4.126 shows three             Problem 26. In Fig. 4.127, the straight
infinitely long straight parallel current         wire AB is fixed while the loop PQRS is free
carrying conductors. Find the (i) magnitude       to move under the influence of the electric
and direction of the net magnetic field at        currents flowing in them. In which direction
point A lying on conductor 1, (ii) magnetic       does the loop begin to move? Give reason
force on conductor 2. [CBSE F 17]                 for your answer.
                                                  [CBSE D 97 ; OD 99]
                 Fig. 4.126
   Solution, (i) Magnetic field at point A
due to current 31 in conductor 2,
                                                                  Fig. 4.127
                       acting into the plane
                                                      Solution. The currents in QR and PS
of the paper                                      have no effect on AB. There is a force of
     Magnetic field at point A due to current     attraction between AB and PQ and a force
4I in conductor 3,                                of repulsion between AB and SR But PQ is
                                                  closer to AB than SR, so force of attraction
                           acting out of the      is stronger than the force of repulsion. So
plane of the paper                                the loop begins to move towards AB.
   Net field at point A,                              Problem 27. Write the expression for
                                                  the magnetic moment ( ⃗⃗ ) due to a planar
                                                  square loop of side carrying a steady
                                                  current I in a vector form.
   acting into the plane of paper
                                                      As shown in Fig. 4.128, this loop is
(ii) Force exerted by conductor 1 on              placed in a horizontal plane near a long
     conductor 2,                                 straight conductor carrying a steady
                                                  current J1 at a distance l. Give reasons to
                                                  explain that the loop will experience a net
                                                  force but no torque. Write the expression
away from conductor 1/towards conductor           for this force acting on the loop.
3.                                                        [CBSE D 10]
Force exerted by conductor 3 on conductor
   2, μ0 2(37)(47)
planar square loop of side I and carrying             carrying current I, when placed in a
current I is given by                                 magnetic field ⃗ , is given by
    In vector form, ⃗⃗       ̂                        ⃗
where ̂ is a unit vector normal to the plane          Obviously, torque will become zero if
of the loop in the sense given by right hand      the area vector      is in the direction of
thumb rule.                                                      ⃗
                                                  external field . Hence in position of stable
    Force of attraction on conductor AB,          equilibrium, the current loop will orient itself
                                                  with its plane perpendicular to the direction
                                                  of ⃗ (because the direction of is normal
                                                  to the plane of current loop).
    Force of repulsion on wire CD,                    In this orientation, the magnetic field
                                                  produced by the loop is in the same
                                                  direction as external field, both normal to
                                                  the plane of the loop, thus giving rise to
     net                                          maximum flux of the total field.
                                                      (c) The loop will assume circular shape
    As Fa > Fr, the net force on loop ABCD
                                                  with its plane perpendicular to the
is attractive.
                                                  magnetic field so as to maximise magnetic
   As the area vector is parallel to the          flux. This is because for a given perimeter,
magnetic field, θ = 0°, so torque on the          a circle encloses greater area than any
square loop,                                      other shape.
                                Thus        the       Problem 29. Why is the magnetic field
torque acting on the loop is zero.                radial in a moving coil galvanometer?
                                                  Explain how it is achieved.
    Problem 28. (a) A current carrying
circular loop lies on a smooth horizontal                                        [CBSE OD 15C]
plane. Can a uniform magnetic field be set
                                                        Solution. Radial magnetic field. By
up in such a manner that the loop turns
                                                  using pole pieces of a magnet and placing
around itself (i.e., turns about the vertical
                                                  a soft iron cylindrical core between the
axis)?
                                                  concave poles, we get a magnetic field with
     (b) A current carrying circular loop is      its lines of force pointing along the radii of a
located in a uniform external magnetic            circle. Such a field is called a radial field.
field. If the loop is free to turn, what is its   The plane of a coil rotating in such a field is
orientation of stable equilibrium? Show that      always parallel to the field, as shown in Fig.
in this orientation the flux of the total field   4.129.
(external field + field produced by the loop)
is maximum.
                                 [CBSE F 13]
     (c) A loop of irregular shape carrying
current is located in an external magnetic
field. If the wire is flexible, why does it
change to a circular shape?
                                                                    Fig. 4.129
                                     [NCERT]
                                                      Need of radial field. The current I
    Solution, (a) No, that would require to       through a galvanometer coil is given by
act in the vertical direction. But
⃗ and for the horizontal loop acts in the
vertical direction, so acts in the plane of
                                                      Because of the presence of factor sin θ,
the horizontal loop.
                                                  the deflection α of the galvanometer coil is
(b) The torque on a loop of area      and         not quite proportional to the current I, so
that the instrument is not a linear one. To       the component across which the p.d. is
make its scale linear, the field is made          measured. It must have very high
radial. Then θ = 90°, so that                     resistance so that a very small current
                                                  passes through it and the p.d. across that
                         or                       component is not affected. So a
                                                  galvanometer is converted into a voltmeter
    Problem 30. Describe the working              by connecting a high resistance in series
principle of a moving coil galvanometer.          with it.
Why is it necessary to use (i) a radial
                                                     Problem 32. Compare a voltmeter and
magnetic field and (ii) a cylindrical soft iron
                                                  an ammeter.
core in a galvanometer?
                                                      Solution.
    Can a galvanometer as such be used
for measuring the current? Explain.
                                                           Ammeter              Voltmeter
                                [CBSE D 17]
    Solution. Principle of a moving coil           1. It    is    a     low It   is    a     high
galvanometer: A current-carrying coil                 resistance device resistance device
placed in a magnetic field experiences a              used to measure used to measure
current dependent torque, which tends to              electric current.     potential difference.
rotate the coil and produces angular
                                                   2. It is obtained by It is obtained by
deflection.
                                                      connecting a low connecting a high
(a) Radial magnetic field maximizes the               resistance      in resistance in series
    torque on the coil and provides linear            parallel   with  a with               a
    current scale.                                    galvanometer.      galvanometer.
(b) A cylindrical soft iron core makes the
    field radial. This increases the strength      3. It    is  always It     is         always
    of the magnetic field and hence                   connected     in connected             in
    increases the sensitivity of the                  series.          parallel.
    galvanometer.
                                                   4. It is not possible to It is possible to
   A galvanometer as such cannot be                   decrease the range decrease the range
used to measure current in a circuit due to           of       a     given of      a     given
two reasons:                                          ammeter.              voltmeter.
(i) A galvanometer can detect only small
    currents and its coil is likely to be             Problem 33. Define current sensitivity
    damaged by currents of (mA/A) range.          and voltage sensitivity of a galvanometer.
                                                  Increasing the current sensitivity may not
(ii) For measuring currents, it has to be         necessarily increase the voltage sensitivity
     first calibrated.                            of a galvanometer. Justify.
    Problem 31 Why is it that while using a                                  [CBSE OD 09,11]
moving coil galvanometer as a voltmeter a
high resistance in series is required                 Solution. Current sensitivity of a
whereas in an ammeter a shunt is used?            galvanometer is the deflection produced
                                                  per unit flow of current while voltage
                 [CBSE OD 10, 12 ; D 13C]         sensitivity is the deflection produced per
    Solution. In order that the insertion of      unit applied potential difference.
an ammeter in series in a circuit does not            Current sensitivity,
affect the current in the circuit, an ammeter
must have least possible resistance. So a             Voltage sensitivity,
galvanometer is converted into ammeter
by connecting a small resistance in parallel          If the current sensitivity is increased by
with it.                                          increasing the number of turns N, the
    A voltmeter is connected in parallel with     resistance R will also increase. So, the
                                                                     Fig. 4.134
                                                   (i) Trace its trajectory.
                                                   (ii) Does the charge gain kinetic energy as
                                                   it enters the magnetic field? Justify your
                                                   answer. [CBSE D 09]
                                                       Solution. (i) Charge c begins to move in
                      Fig. 4.132                   a circular orbit in the XY-plane as shown in
                                                   Fig. 4.135.
    Solution. For the statement of
Biot-Savart law, refer to point 2 of                   (ii) As the magnetic force acts
Glimpses.                                          perpendicular to the velocity     of the
                                                   charge q, it does no work and hence there
                  ⃗                                is no gain in the kinetic energy of the
                                                   charge q.
                                                                     Fig. 4.135
                      Fig. 4.133                        Problem 3. A long wire is bent into a
        Current element        is along            circular coil of one turn and then into a
Z-direction and position vector is along           circular coil of smaller radius having n
Y-direction, so                                    turns. If the same current passes in both
                                                   the cases, find the ratio of the magnetic
                              ̂            ̂       fields produced at the centres in the two
              ⃗
                                                   cases.
                                                      Solution. Let l be the length of the wire.
                                       ̂
                                                   When the wire is first bent in the form of
                      ̂                            one turn circular coil,
                          ̂        ̂
    Hence the magnetic field at point P is                               or
directed along negative X-direction as
shown in Fig. 4.133.
   Problem 2. A charge ‘q' moving along
or
                                                                         Fig. 4.136
     Problem 4. Show that the magnetic                  Clearly,
field along the axis of a current carrying
circular coil of radius r at a distance x from
the centre of the coil is smaller by the                                          ⁄
fraction 3x2/2r2 than the field at the centre               But
                                                                                          ⁄
of the coil carrying current.                                                     √               ⁄   √
                                                                                      √
    axial
                                  ⁄                         Similarly,
   centre
* +
                       [                  ]                         (directed outward)
                           √
                                                           As the net magnetic field at P is zero, so
   Magnetic field at point P due to the
                                                               = ,
conductor along y-axis is
                                                                                 or
                              *               +
                                                                Problem 8. As shown in Fig. 4.140, a
                          [               ]                cell is connected across two points A and B
                              √                            of a uniform circular conductor. Prove that
     According to right hand thumb rule,                   the magnetic field at its centre O will be
both B1 and B2, act normally into the plane                zero.
of paper. Hence the resultant magnetic
                                                           [Punjab 99C]
field at point P is
B= B1 + B2
          [                       (   )   (       )]
              √
                      √
                  *                           +
                      *√                          +                         Fig. 4.140
    Problem 7. Two insulating infinitely                        Solution. Let the lengths of the two
long conductors carrying currents 1↑ and l2                circular segments ACB and ADB be l1 and
lie mutually perpendicular to each other in                l2, and p be the resistance per unit length.
the same plane, as shown in Fig. 4.138.                    Then
Find the locus of the point at which the net                  Resistance of segment ACB,
magnetic field is zero.
                                                              Resistance of segment ADB, =
Fig. 4.138
    Solution. According to right hand thumb
rule, the magnetic fields due to the two                      Suppose f and I2 are the currents in
conductors can vanish only in regions 1                    segments ACB and ADB respectively. As
and III.                                                   the two segments are connected in
                                                           parallel, so the potential differences across
    Let the magnetic field be zero at point P              them must be equal.
(x, y).
                                                                                       or
                                                              or
                                                               Magnetic field at the centre of an arc of
                                                           length l carrying current I is
̂ ̂ ̂
⃗ ̂ ̂
                                                                             Fig. 4.144
                                                                         ⃗       ⃗    ⃗
                                                         or
                  Fig. 4.143
    Solution. As the point O lies on the
straight part AB, so                                            normally outward.
BAB =0                                                       Problem 13. If the current density in a
                                                         linear conductor of radius a varies with r
                  acting normally outward                according to the relation : J = kr2, where k is
                                                         a constant and r is the distance of a point
                                                         from the axis of the conductor. Find the
                                                         magnetic field at a point distance r from the
acting normally outward                                  axis when (i) r < a and (ii) r > a.
    Total magnetic field at the centre O,
or                        ,       acting      normally
outward.                                                                     Fig. 4.145
    Problem 12. Two infinitely long wires                    Solution, (j) As shown in Fig. 4.145,
carry equal current I. Each wire follows a               consider an elementary ring of radius r and
90° arc along the circumference of the                   thickness dr with its centre on the axis of
same circle of radius R, as shown in Fig.                the conductor.
4.144. Find the magnetic field at the centre             Area of the elementary ring,
O.
                                                         Current passing through the elementary
    Solution, ⃗       ⃗       ⃗           ⃗         ⃗    ring,
⃗     ⃗
    But ⃗         ⃗                   ⃗         ⃗
                                                         Total current enclosed by the closed loop
                                                         of radius r,
                                                                                                                   ̂         ̂                       ̂           ̂
                                                                            ̂                  ̂
                                                            Or r                                                                           ̂                 ̂
                        Fig. 4.147
                  lso,
Or Fig. 4.149
⁄ ⁄
(i) At x - 0,
                                ⁄
                                                                            √
                                     √                                                     √
                                                                  √
acting vertically downwards.
                                                      (iii) At x = 2 a,
     Magnetic field at point O' due to current
I / 2 in coil Y,
                                                                                       ⁄
                                                                        √
acting vertically upwards.                                      ⁄
                                                                                       √
(i) Can both be accelerated at the same               Radius of the circular path of a charged
    cyclotron frequency? Give reason to           particle in a magnetic field is given by
    justify your answer.
(ii) When they are accelerated in turn,                                            √
                                                                          √
     which of the two will have higher
     velocity at the exist slit of the dees?
                                                             √             √
                             [CBSE OD 13]             ∴
                                                           Hence                   √
    Hence, α-particles and protons will not
accelerate at the same cyclotron                      Problem 20. A neutron, a proton, an
frequency. In fact, cyclotron frequency for       electron and an a-particle enter a region of
protons is twice the cyclotron frequency of       constant magnetic field with equal
α-particles.                                      velocities. The magnetic field is along the
                                                  inward
   Also, maximum velocity acquired by
accelerated particles is given by
or
proton
             particle
                                                                     Fig. 4.150
    Hence, the exit velocity of protons is
twice the exit velocity of α-particles.           normal to the plane of the paper. The
                                                  tracks of the particles are labelled in Fig.
    Problem 19. A proton, a deutron and           4.150. Relate the tracks to the four
an alpha particle having the same kinetic         particles.
energy are allowed to pass through a                                                         [
uniform magnetic field perpendicular to           IIT 84]
their direction of motion. Compare the radii
of their circular paths.                              Solution, (i) For a neutron, q = 0, so F =
                                                  qvB sin θ = 0. That is, a neutron goes
                                [AIEEE 12]        undeflected through the magnetic field. So
   Solution. If mp, md and mα are the             the track C corresponds to a neutron.
masses of proton, deutron and α-particle              (ii) According to Fleming's left hand
respectively, then                                rule, a negative charged particle such as
   md = 2 mp and ma = 4 mp                        an electron will be deflected towards right.
                     and                          So the track D corresponds to an electron.
                                                      (iii) According to Fleming's left hand
   Kinetic energy,                                rule, a positively charged particle such as
                                                  an α-particle or a proton will be deflected
                                                  towards left. Its radius of curvature is given
by
i.e.,
                      i.e.,
                                                      Let B' = nB, be the new magnetic field
   Thus the track B corresponds to an
                                                  in region II. If 1 is the radius of the new
a-particle and track A to a proton.
                                                  circular path, then
    Problem 21. A fine pencil of β
-particles, moving with a speed v, enters a
region (region I), where a uniform electric
and a uniform magnetic field are both                Hence, the radius of the circular path
present. These β -particles then move into        would decrease by a factor n.
region II where only the magnetic field (out
of the two fields present in region I), exists.        Problem 22. Two long wires carrying
The path of the β -particles, in the two          currents L and I2 are arranged as shown in
regions, is as shown in Fig. 4.151.               Fig. 4.152. The one carrying current f is
                                                  along the x-axis. The other carrying current
                                                  f is along a line parallel to the y-axis given
                                                  by x = 0 and z-d. Find the force exerted at
                                                  02 because of the wire along the X-axis.
[Exemplar Problem]
                  Fig. 4.151
(i) State the direction of the magnetic field.
(ii) State the relation between 'E' and 'B' in
     region 1.
(iii) Drive the expression for the radius of
      the circular path of the β -particle in
      region II.
                                                                   Fig. 4.152
    If the magnitude of magnetic field, in
region II, is changed to n times its earlier           Solution. At 02, the magnetic field due
value, (without changing the magnetic field       to f is along the y-axis.
in region I) find the factor by which the            The second wire also lies along y-axis
radius of this circular path would change.        and hence the force is zero.
[CBSE Sample Paper 13]
    Solution, (i) As per Fleming's left hand
rule, the magnetic field acts perpendicular       Problem 23. Find the magnitude of the
to the plane of paper and is directed             force on each segment of the wire as
inwards.                                          shown in Fig. 4.153, if a magnetic
Here l = 10 cm = 0.10 m,
towards left.
                                                                         √            Nm
                                                            Problem 29. A length L of a wire carries
                                                        a current I. Show that if the wire is formed
                    Fig. 4.156
                                                        into a circular coil, the maximum torque in a
    Solution. Here                                      given magnetic field is developed when the
                  and                    ⃗⃗       ⃗     coil has one turn only and the maximum
                                                        torque has the value,
    (a)         ̂        ̂                    ̂
                                                        Solution. Torque on a current loop,
    ∴ Magnitude of torque = BIA
    (b)              ⃗          ̂    ⃗                  If the circular coil has N turns, each of
                                                        radius r, then L = 2πr N L
    ∴ Magnitude of torque = 0.
     Problem 28. A coil in the shape of an
equilateral triangle of side 0.02 m is
suspended from a vertex such that it is                    Area of the coil,
hanging in a vertical plane between the
pole pieces of permanent magnet
producing a horizontal magnetic field 5 ×
10“ 2 tesla. Find the couple acting on the                 Hence
coil when a current of 0.1 A is passed
through it and the magnetic field is parallel               Clearly, torque will be maximum when
to its plane.                                           sin θ is maximum and N is minimum i.e.,
                                                        sin θ =1 and N =1. Then
                                         [Roorkee 91]
    Solution. Area of the triangle,
A = base × height = a × a sin 60°                           Problem 30. In the circuit shown in Fig.
                         √                              4.158, a voltmeter reads 30 V, when it is
= × 0.02 × 0.02 ×                = √3 × 10-4m2 2        connected across 400 Ω resistance.
                                                        Calculate what the same voltmeter will
                                                        read, when connected across the 300 Ω
                                                        resistance. [IIT 80; Punjab 95]
Fig. 4.158
    Solution. Let Ry be the resistance of the
voltmeter. Then the resistance R of the
parallel combination of RV and R1 will be
or
   On solving,                                    or
RV =1200 Ω
                                                  or
   When this voltmeter is connected
across R2, the combined resistance will be
                                                  or
                                                       Solution,
    Reading of the voltmeter = P.D. across
R' = I' R'
                                                      Problem 33. If a galvanometer is
                                                  connected in series with a high resistance
                               V
                                                  so that potential drop across the
    Problem 31. A voltmeter of resistance         galvanometer is 1/nth of the total applied
RV and an ammeter of resistance RA are            voltage, then show that the combined
connected in a circuit to measure, a              resistance of the galvanometer and the
resistance R as shown in Fig. 4.159. The          series resistor is n times the resistance of
ratio of the meter readings gives an              the galvanometer i.e., RV = nRg.
apparent resistance R'. Show that R and R'        Solution. Here               or
are related by the relation
                                                       Required Resistance,
                R
                    V   = R' + Rg = nRg                     …(ii)
    Problem 34. A galvanometer of.                         On dividing (i) by (ii)
resistance G is converted into a voltmeter
to measure upto V volts by connecting a                                   or
resistance R1 in series with the coil. If a
resistance R2 is connected in series with it,
then it can measure upto V/2 volts. Find the
resistance, in terms of R1 and R2, required                Let R3 be the resistance required for
to be connected to convert it into a                    conversion into voltmeter of range 2 V.
voltmeter that can read upto 2 V. Also find             Then,
the resistance G of the galvanometer in                                                                   ...(iii)
terms of R1 and R2. [CBSE D 15]
                                                           On dividing (iii) by (i), we get
    Solution.
    For   voltmeter           of   range       V,                          or
            …
                                                           ∴
    For voltmeter of range                              = 3R1-2R2
   normal to the field. Explain why the               Ans. (a) N = 30, r = 8.0 cm = 0.08 m, I =
   path of the electron is a circle.              6.0 A,
   Determine the radius of the circular
                                                  B = 1T, 0 = 60°
   orbit. Given that e = 1.6 × 10-19 C, me =
   9.1 × 10-31 kg.                                    Magnitude of counter torque
   Ans. The perpendicular magnetic field          = Magnitude of deflecting torque = NIB A
exerts a force on the electron                    sin θ
perpendicular to its path. This force             = 30 × 6 × 1 × (3.14 × 0.08 × 0.08) sin 60°
continuously deflects the electron from its
path and makes it move along a circular           = 30 × 6 × 3.14 × 64 × 10-4 × 0.866 = 3.1
path.                                             Nm.
   ∴ Magnetic force on the electron =                  (b) No, the answer would not change
Centripetal force                                 because the above formula for the torque
                                                  is true for a planar loop of any shape.
                                                      4.14. Two concentric circular coils X
                                                  and Y of radii 16 cm and 10 cm
                                                  respectively lie in the same vertical plane
                                                  containing the north-south direction. Coil X
   Now                G              T            has 20 turns and carries a current of 16 A;
         ms                                       coil Y has 25 turns and carries a current of
                                                  18 A. The sense of the current in X is
                                                  anticlockwise, and in Y clockwise, for an
                                                  observer looking at the coils facing west.
                      m       cm                  Give the magnitude and direction of the net
                                                  magnetic field due to the coils at their
4.12. In Exercise 4.11, obtain the                centre.
   frequency of revolution of the electron
   in its circular orbit. Does the answer
   depend on the speed of the electron?
   Explain.
    Ans. Frequency of revolution of the
electron in its circular orbit,
                                         T                                     approximately
                                T
                                                     Such an arrangement used to produce
    As the current in the coil Y is clockwise,    a nearly uniform magnetic field over a
the field By is directed towards west. Since      small region is known as Helmholtz coils.
By > Bx, therefore, the net field is directed         Ans. (a) Given
towards west and its magnitude is
                                             T                                      ⁄
4.15.   A magnetic field of 100 G (1 G =
   10-4T) is required which is uniform in a           At the centre of the coil, x = 0, so
   region of near dimension about 10 cm
   and area of cross-section about 10-3m2.
   The maximum current-carrying
   capacity of a given coil of wire is 15 A           This is the standard result for the field
   and the number of turns per unit length        at the centre of the coil.
   that can be wound round a core is at              (b) As shown in Fig. 4.161, consider a
   most 1000 turns m-1. Suggest some              small region of length 2d about the
   appropriate design particulars of a            midpoint O between the two coils.
   solenoid for the required purpose.
   Assume the core is not ferromagnetic.
    Ans. Here B = 100 G = 10-2 T, I = 15 A,
n = 1000 turns m-1
     Magnetic field inside a solenoid,
∴ Fig. 4.161
    The solenoid may have length 50 cm            Therefore, the magnetic field at the point P
and area of cross- section 5 × 10_2m2 (five          due to coil 1, μ∩m2N
times the given value) so as to avoid edge
effects, etc.
    4.16. For a circular coil of radius R and                                    acting along
                                                                           ⁄
N turns carrying current I, the magnitude of
the magnetic field at a its axis at a distance
x from its centre is given by,                    Magnetic field at the point P due to coil 2,
                  ⁄
                                                                                 acting along
     (a) Show that this reduces to the                                     ⁄
familiar result for field at the centre of the
coil.                                             Total magnetic field at the point P will be B
                                                     =
    (b) Consider two parallel co-axial
circular coils of equal radius R, and number
of turns N, carrying equal currents in the                                                     ⁄
same direction, and separated by a
                                                               ⁄
                               (                           )                                               T
                                                                   ]
                                                                                 (c) The field in the empty space
[Neglecting d2, as d << R]
                                                                             surrounded by the toroid is also zero.
                                                                             4.18.    Answer the following questions :
                           [                           ⁄
                       ⁄                                                     (a) A magnetic field that varies in
                               (                   )                             magnitude from point to point but has a
                                                                                 constant direction (east to west) is set
                                                                   ]             up in a chamber. A charged particle
                                                               ⁄                 enters the chamber and travels
                                       (                   )                     undeflected along a straight path with
                                                                                 constant speed. What can you say
             ⁄                                                                   about the initial velocity of the particle?
                 *(                )               (                   ) +
                                                                             (b) A charged particle enters an
                                                                                 environment of a strong and
                 ⁄
                      [(               )           (                   )]        non-uniform magnetic field varying
                                                                                 from point to point both in magnitude
 [Expanding by binomial theorem and                                              and direction and comes out of it
neglecting higher powers of d / R]                                               following a complicated trajectory.
                                                                                 Would its final speed equal the initial
or                                                                               speed if it suffered no collisions with the
                                                                                 environment ?
    Magnetic field will also be same at the
point Q. In fact, it will be uniform over the                                (c) An electron travelling west to east
small region of length 2 d around the                                            enters a chamber having a uniform
midpoint O.                                                                      electrostatic field in a north to south
                                                                                 direction. Specify the direction in which
4.17. A        toroid     has      a      core                                   a uniform magnetic field should be set
   (non-ferromagnetic) of inner radius 25                                        up to prevent the electron from
   cm and outer radius 26 cm around                                              deflecting from its straight line path.
   which 3500 turns of a wire are wound. If
   the current in the wire is 11 A, what is                                    Ans. (a) The force on a charged particle
   the magnetic field (a) outside the toroid                                 moving in a magnetic field is given by
   (b) inside the core of the toroid (c) in the                                  F = qvB sin θ
   empty space surrounded by the toroid?
                                                                                  The force on a charged particle will be
    Ans. Here, I = 11 A, total number of                                     zero or the particle will remain undeflected
turns = 3500                                                                 if sin θ = 0 or θ = 0°, 180°
     Mean radius of toroid,                                                  i.e., initial velocity v is either parallel or
                                                                             antiparallel to B .
                           cm                                           m
                                                                                (b) Yes, a magnetic field exerts force
Total length (circumference) of the toroid                                   on a charged particle in a direction
                                         m                                   perpendicular to its direction of motion and
                      m      mm                                                       Ckg
    Thus the electron follows a circular            = 4.8 × 107 C kg-1
trajectory of radius 1 mm normal to the field
B.                                                       Now for deutrons,
proton and one neutron. The answer is not              long and 1.5 cm apart? Is the force
unique because we have determined only                 attractive or repulsive?
the ratio of charge to mass. Other possible
                                                                                   [Haryana 01]
answers are He2+ and Li3+, etc.
                                                   Ans.                                cm
    4.21 A straight horizontal conducting
                                                            m l = 70 cm = 0.70 m
rod of length 0.45 m and mass 60 g is
suspended by two vertical wires at its ends.           The force per unit length between the
A current of 5.0 A is set up in the rod            wires is
through the wires.
   (a) What magnetic field should be set                                                    Nm
up normal to the conductor in order that the                             Nm
tension in the wires is zero?
                                                       Total force between the wires,
                                 [CBSE D 15C]
                                                       F= f×l = 1.2 × 0.70 = 0.84 N
    (b) What will be the total tension in the
wires if the direction of current is reversed,        As the currents in the two wires are in
keeping the magnetic field same as                 opposite directions, the force is repulsive.
before? (Ignore the mass of the wires) g =             4.23. A uniform magnetic field of 1.5 T
9.8 ms-2.                                          exists in a cylindrical region of radius 10.0
        Ans Here l = 0.45 m, m - 60 g = 0.06 kg,   cm, its direction being parallel to the axis
                                                   along east to west. A wire carrying current
7 = 5.0 A, g = 9.8 ms-2                            of 7.0 A in the north to south direction
    (a) Tension in the supporting wires will       passes through this region. What is the
be zero when the weight of the rod is              magnitude and direction of the force on the
balanced by the upward force IIB of the            wire if
magnetic field.                                    (i) the wire intersects the axis,
i.e.,          I IB = mg                           (ii) the wire is turned from N-S to north
                                                        east or north west direction,
                               T        T
                                                   (iii) the wire in the N-S direction is lowered
                                                         from the axis by a distance of 6.0 cm ?
                                                   Ans. Here B = 1.5 T, I = 7.0 A (i) As shown
                                                   in Fig. 4.163, length of wire in cylindrical
                                                   region
                                                   = diameter AB of cylindrical region = 20 cm
   According to Fleming's left hand rule,          - 0.20 m
the magnetic field should be applied
normally into the plane of paper so as to
exert an upward magnetic force on the rod.
   (b) If the direction of current is
reversed, the magnetic force will act in the
downward direction. Hence the total
tension in the wires will be
T = 2 × the weight of the rod
                                                                    Fig. 4.163
= 2 × 0.06 × 9.8 N = 1.176 N.
                                                        As the wire lies in N-S direction and
4.22. The wires which connect the battery          field acts along E-W direction, so θ = 90°
   of an automobile to its starting motor
   carry a current of300 A (for a short                ∴ Force on wire,
   time). What is the force per unit length
                                                       F = IBl sin θ = 7 × 1.5 × 0.20 × 1 = 2.1 N
   between the wires if they are 70 cm
                 Fig. 4.164
Force on wire,
                                                                        Fig. 4.165
This force acts in the vertically downward
                                                  and 5 cm carries a current of 12 A. What is
direction.
                                                  the torque on the loop in the different cases
    (iii) As shown in Fig. 4.159, when the        shown in Fig. 4.165? What is the force on
wire is lowered by 6.0 cm, length of the          each case? Which case corresponds to
wire in the magnetic field = 2x                   stable equilibrium?
But               √             cm                    Ans. Here B = 3000 G = 3000 × 10-4 =
      m                                           0.3 T,
∴ 2x = 0.16, θ = 90°                                  A = 10 × 5 = 50cm2 = 50 × 10-4m / = 12
                                                  A
Force on wire, F = IIB = 7 × 0.16 × 1.5 =
1.68 N                                                Magnetic moment,
                                                                    ̂    ̂                 ̂
                                                                                      -2
                                                      Thus a torque of 1.8 × 10            Nm acts
                                                  along negative Y-axis.
                                                      (b) Here ⃗⃗             ̂ m ⃗            ̂T
                                                      Clearly, m and B are same as in case
                                                  (a). In this case also, a torque of 1.8 × 10-2
                                                  Nm acts along negative Y-axis.
                                 ⃗⃗        ⃗
                 ̂           ̂                          N
                                                        ̂ m
    Thus a torque of 1.8 × 10                      2
                                                       Nm acts             For given wire,
along negative X-axis.                                                                   m                     m
    (d) This case is similar to case (c). But
here the direction of the torque is 60°                                                        N                     N
anticlockwise with negative X-direction i.e.,
                                                                   4.26.   A solenoid 60 cm long and of
240° with positive X-direction.
                                                                      radius 4.0 cm has 3 layers of windings
    (e) Here ⃗⃗                       ̂ m ⃗             ̂T            of 300 turns each. A 2.0 cm long wire of
                                                                      mass 2.5 g lies inside the solenoid near
            ⃗⃗           ⃗             ̂       ̂
                                                                      its centre normal to its axis ; both the
    (f) Here ⃗⃗                       ̂ m ⃗              ̂T           wire and the axis of the solenoid are in
                                                                      the horizontal plane. The wire is
           ⃗⃗        ⃗                 ̂       ̂                      connected through two leads parallel to
                                                                      the axis of the solenoid to an external
   The net force on the loop is zero in
                                                                      battery which supplies a current of 6.0
each case.
                                                                      A in the wire. What value of current
     Case (e) corresponds to stable                                   (with appropriate sense of circulation)
equilibrium, because here ⃗⃗ in is parallel                           in the windings of the solenoid can
to ⃗                                                                  support the weight of the wire? g = 9.8
                                                                      ms-2.
    Case (f)          corresponds to unstable
equilibrium,         because here ⃗⃗ in is                             Ans. Let I be the current in the windings
antiparallel to      ⃗.                                            of the solenoid which can support the
                                                                   weight of the wire. The magnetic field
4.25.   A circular coil of 20 turns and                            inside the solenoid along its axis will be
   radius 10 cm is placed in a uniform
   magnetic field of 0.10 T normal to the                          B - μ Q nl
   plane of the coil. If the current in the coil                                      Total number of turns
                                                                           Here,      Length of the solenoid
   is 5.0 A, what is the (a) total torque on
   the coil,
                                                                                                      turns m
(b) total force on the coil,
(c) average force on each electron in the
    coil due to the magnetic field ?                                                                 tesla
    (The coil is made of copper wire of                                This field acts perpendicular to the
cross-sectional area 10-5m2, and the free                          current carrying wire, therefore, the
electron density in copper is given to be                          magnetic force on the wire will be
about (1029 m-3).
   Ans. N = 20, r = 10 cm = 0.10 m, B =
0.10T,                                                                                                         newton
1 = 5.0 A, 0 = 0°                                                      The current 7 would support the wire if
                                                                   the above force equals the weight of the
    (a) Torque on the coil,                                        wire,
  = NIB A sin θ = 0                                           [∴   i.e.,
θ = 0°]
    (b) Magnetic forces on the opposite
arms of coil are equal and opposite, and
act in the same plane ; hence the total
force on the coil is zero.                                                 4.27. A galvanometer coil has a
                                                                                = 0.010Ω =10
= 6000- 12 = 5988 Ω                               mΩ
    By connecting a resistance of 5988 in            By connecting a shunt of resistance 10
series with the given galvanometer, we get        mΩ across the given galvanometer, we get
a voltmeter of range 0 to 18 V.                   an ammeter of range 0 to 6 A.
                                    TEXT BASED EXERCISES
                  TYPE A : VERY SHORT ANSWER QUESTIONS 1 mark each)
1. State Oersted's observation.                        element (i)      minimum      and     (ii)
                                                       maximum?
2. State Biot-Savart's law.
                                                  12. Figure 4.166 shows a circular loop
                                    [ISCE 94]
                                                      carrying a current I.
3. Mathematically, Biot-Savart law may
                                                  Show the direction of the magnetic field
   be expressed as
                                                  with the help of lines of force. [CBSE D 04]
27. An electron beam is moving vertically         36. Write the condition under which an
    downwards. If it passes through a                 electron will move undeflected in the
    magnetic field which is directed from             presence of crossed electric and
    south to north in a horizontal plane,             magnetic fields. [CBSE F 13 ; OD 14C]
    then in which direction the beam would        37. A straight conductor AB of a circuit lies
    be deflected ?     [CBSE D 96C]                   along the X-axis from x = - a / 2 to x = +
28. What will be the path of a charged                a / 2 and carries a current 1. What is the
    particle moving perpendicular to a                magnetic field due to this conductor AB
    uniform magnetic field ?                          at a point x = + a ?
   magnitude of the field and does not            53. What is the value of net force acting on
   depend upon the velocity. What is the              a current carrying (z) rectangular and
   nature of the field?                               (ii) circular loop, placed in a uniform
                                                      magnetic field ? What do you expect
42. What is the force that a conductor ⃗⃗⃗ ,          about the torque in each case ?
    carrying a current I experiences when
    placed in a magnetic field ⃗ . What is        54. Write an expression for the torque
    the direction of the force?                       acting on a current carrying coil located
                                                      in a uniform magnetic field.
                              [CBSE OD 90]
                                                  55. Write an expression for the magnitude
43. An     electroii   beam    is   moving            of the torque acting on a current
    horizontally in a tube. The vertical              carrying coil placed in a uniform radial
    component of earth's magnetic field is            magnetic field.
    directed downwards. In which direction
    will the electron beam be deflected?          56. Under what circumstances will a
                                                      current carrying loop not rotate in the
44. A charged particle moves in a uniform             magnetic field ?
    magnetic field at right angles to the
    direction of the field. Which of the          57. State the principle of working of a
    following quantities will change : speed,         moving       coil      galvanometer.
    velocity, momentum, kinetic energy,                [CBSE D 15, 16]
    displacement ?                                58. What do you mean by the figure of
45. In which orientation is the force                 merit of a galvanometer ?
    experienced by a current-carrying             59. Define the current sensitivity of a
    conductor placed in a magnetic field              moving coil galvanometer and state its
    maximum ?                                         SI unit.
46. A current carrying conductor does not                  [CBSE D 17 ; OD 13, 13C, 16, 17C]
    tend to deflect in a magnetic field. What
    conclusion can be drawn from it ?             60. Write two factors by which the current
                                                      sensitivity  of    a   moving      coil
47. Name the rule that gives the direction            galvanometer can be increased.
    of force on a current-carrying conductor
    placed perpendicular to the magnetic                          [CBSE D 01 ; F 08 ; OD 16]
    field.                                        61. Define voltage sensitivity of a moving
48. Write an expression for the force                 coil galvanometer. Give its SI unit.
    between two parallel short wires                   [Punjab 91]
    carrying currents.                            62. Write two factors by which voltage
49. Two current elements are placed a                 sensitivity of    a    moving  coil
    certain distance apart but not parallel to        galvanometer can be increased.
    each other. Do they exert equal and                                   [CBSE D 01 ; F 08]
    opposite forces on each other ?
                                                  63. What is the nature of the magnetic field
50. What is the direction of force between            in a moving coil galvanometer ?
    two parallel wires carrying currents in             [CBSE D 96 ; OD 96]
    opposite directions ?
                                                  64. State two properties of the material of
51. The force existing between two parallel           the wire used for suspension of the coil
    current carrying conductors is F. If the          in a moving coil galvanometer.
    current in each conductor is doubled,               [CBSE OD 01, 06C]
    what is the value of the force between
                                                  65. The current sensitivity of a moving coil
    them ?
                                                      galvanometer is 5 division/mA and
52. Is the force between two parallel                 voltage sensitivity is 20 division/volt.
    current-carrying wires affected by the            Find    the    resistance     of    the
    nature of the dielectric medium                   galvanometer.
    between them ?
66. An electron and a proton, having equal              enters normal to the field direction and
    momenta, enter a uniform magnetic                   the other enters along a direction at 30°
    field at right angles to the field lines.           with the field, what would be the ratio of
    What will be the ratio of curvature of              their angular frequencies ?
    their trajectories ?
                                                                      [CBSE Sample Paper 08]
                  [CBSE Sample Paper 05]
                                                  79. n α-particle and a proton are moving
67. An electron is moving with a velocity v,         in the plane of the paper in a region
    along the axis of a long straight                where there is a uniform magnetic field
    solenoid, carrying a current I. What will        ( ⃗ ) directed normal to the plane of the
    be the force acting on the electron due          paper. If the two particles have equal
    to the magnetic field of the solenoid ?          linear momenta, what will be the ratio of
                  [CBSE Sample Paper 05]             the radii of their trajectories in the field
                                                     ? [CBSE Sample Paper 08]
68. Among alpha, beta and gamma
    radiations, which get deflected by the        80. Why should the spring/suspension wire
    magnetic field ? [CBSE F 04]                      in a moving coil galvanometer have low
                                                      torsional constant ?
69. A solenoid coil of 300 turns/m is
    carrying a current of 5 A. The length of                                      [CBSE OD 08]
    the solenoid is 0.5 m and has a radius        81. The coils, in certain galvanometers,
    of 1 cm. Find the magnitude of the                have a fixed core made of a
    magnetic field inside the solenoid.               non-magnetic metallic material. Why
      [CBSE F 04]                                     does the oscillating coil come to rest so
70. What is the resistance of an ideal                quickly in such a core ?
   ammeter ?                                                                      [CBSE D 08C]
71. What is the resistance of an ideal            82. A long straight wire carries a current I
   voltmeter ?                                        along the positive y-direction. A particle
72. Why should an ammeter have a high                 of charge +Q is moving with a velocity
    current carrying capacity ?                         along the .t-axis. In which direction
                                                      will the particle experience a force ?
73. Why should a voltmeter have a low
    current carrying capacity ?                                                     [CBSE F 13]
74. What is the effective resistance of an        83. Two particles A and B of masses m and
    ammeter if a shunt of resistance Rs is            2m have charges q and 2q
    used across the terminals of a                    respectively. Both these particles
    galvanometer of resistance R ?                    moving with velocities v1 and v2
                                                      respectively in the same direction enter
75. Suppose a shunt of resistance 0.01 Ω              the same magnetic field B acting
    is connected across a galvanometer,               normally to their direction of motion. If
    what can be said about the resistance             the two forces FA and FB acting on them
    of the resulting ammeter ?                        are in the ratio 1 : 2, find the ratio of
76. A student wants to increase the range             their velocities.
    of an ammeter from 1 mA to 5 mA.                                             [CBSE D 11 C]
    What should be done to the shunt
    resistance ?                                  84.      beam of α particles projected along +
                                                        x-axis, experiences a force due to
77. What is the direction of the force acting           magnetic field along the + y-axis. What
    on a charged particle q, moving with a              is the direction of the magnetic field ?
    velocity in a uniform magnetic field ⃗              (Fig. 4.167) 4      [CBSE OD 10]
    ?
78. Two identical charged particles moving
    with same speed enter a region of
    uniform magnetic field. If one of these
                    Fig. 4.168
86. A narrow stream, of protons and                                   Fig. 4.169
Answers
1. A magnetic needle brought close to a                 According to this rule if we grasp the
straight current-carrying wire aligns itself            conductor in the right hand so that the
perpendicular to the wire, reversing the                thumb points in the direction of the
direction of current reverses the direction             current, then the magnetic field will be
of deflection.                                          in the direction of the curl of the fingers.
2. According to Biot-Savart law, the               7.                        8. See Fig. 4.8.
   magnetic field due to a current element
   I ⃗⃗⃗ at the observation point whose            9. The magnetic field consists of concentric
   position vector is is given by                  circular lines of force with the conductor at
                                                   their centre and in a plane perpendicular to
                              ⃗⃗⃗                  the conductor.
                ⃗
                                                   10 At the centre of the current loop.
where μ 0 is the permeability of free space.
                                                   11. (i) Magnetic field is minimum (zero)
3.                       Tm                            along the axis of a current element.
19. The force experienced by a charged            33. According to Fleming's left hand rule,
    particle while moving through a region            the magnetic field acts in the
    of magnetic field is called magnetic              +Z-direction.
    Lorentz force.
                                                  34. Radius,                i.e.,
It is given by            ⃗
                                                  As electron has smaller mass than proton,
20.              ⃗ The direction of the force     so it will circulate in a circular path of
      is perpendicular to the plane containing    smaller radius.
      vectors and ⃗ .                             35. Radius of curvature,
21. F = qvB sin θ.
                                                                             √
22. For a stationary charge, v = 0.                                  √                      i.e.,       √
Therefore, F = qvB sin θ = q (0) B sin θ = 0.
23. Zero, because a magnetic force acts                ∴ If the kinetic energy is halved, radius
    perpendicular to the direction of             of curvature is reduced to ⁄√ times its
    velocity or the direction of motion of the    initial value.
    charged particle.                             36. For undeflected beam,
24.        ⃗        ⃗
                                                               or                               or
25. Force on a charge moving parallel or
    antiparallel to the direction of the          37. Zero, because the observation point
    magnetic field is zero F = qvB sin (0° or         lies on the axis of the straight
    180°) = 0.                                        conductor.
26. Fleming's left hand rule gives the            38. Refer answer to Q. 16 on page 4.34.
    direction of force on a charged particle
                                                  39. Time spent by a proton inside the dees
    moving in a magnetic field : Stretch the
                                                      of a cyclotron is independent of both its
    thumb and the first two fingers of the
                                                      speed and radius of its circular path.
    left hand so that they are perpendicular
    to each other. If the forefinger points in    40. The field is magnetic in nature.
    the direction of magnetic field, central
                                                  41. The field is electric in nature.
    finger in the direction of current, then
    the thumb gives the direction of force        42.          ⃗⃗⃗       ⃗
48. The force between two parallel short          The SI unit of current sensitivity is radian
    wires of lengths      and     , separated        ampere-1.
    by distance r and carrying currents I1        60. The current sensitivity of a moving coil
    and I2 respectively, is given by                  galvanometer can be increased by (i)
                                                      increasing the number of turns in the
                                                      galvanometer coil, (ii) decreasing the
                                                      torsion constant of its suspension fibre.
49. Like other forces, these forces also
    obey Newton's third law of action and         61. The voltage sensitivity of a moving coil
    reaction and are, therefore, equal and            galvanometer is defined as the
    opposite.                                         deflection    produced        in    the
                                                      galvanometer when a unit voltage is
50. The direction of force is perpendicular
                                                      applied across its coil.
    to the two wires and is outwards, so
    that the two wires repel each other.          Voltage sensitivity =
51. The value of force is 4F. This is             The SI unit of voltage sensitivity is radian
    because force between two parallel               volt-1.
    current    carrying   conductors is
    proportional to the product of the            62. The voltage sensitivity of a moving coil
    currents through them.                            galvanometer can be increased by
52. No. This interaction is between the           (i) increasing the number of turns of the
    magnetic fields produced by the two                galvanometer coil
    wires which does not depend on the            (ii) decreasing the torsion constant of the
    nature of the dielectric medium.                   suspension fibre.
53. In each case the net force is zero but        63. Radial magnetic field is used in a
    torque is non-zero.                               moving coil galvanometer.
54. If a coil of area A, turns N and carrying     64. The material used for the suspension
    current 1 is held in a uniform magnetic           wire of a moving coil galvanometer
    field ⃗ , it experiences a torque given by        should have following properties :
       = NIB A sin θ, where θ is the angle
                                                  (i) Small torsion constant k which makes
    between ⃗ and the normal to the plane
                                                      the galvanometer highly sensitive.
    of the loop.
(ii) High tensile strength so that even thin      plane of the vectors       ⃗.
     wire does not break under the weight of
     the suspension coil.                         78. Angular frequency,              . It is
65. Here Is - 5 div mA-3 = 5 × 103div A-1,            independent of m angle θ.
    loop of radius r carrying a steady            (i) inside on the axis and (ii) outside the
    current I. Draw the field lines due to the    combined system.      [CBSE D 14]
    current loop.
       [ISCE 96 ; CBSE D 01C ; OD 14 C]
5. State Biot-Savart law. Deduce the
   expression for the magnetic field at a
   point on the axis of a current carrying
   circular loop of radius ' R', distant V
   from the centre. Hence write the
   magnetic field at the centre of a loop.
     [CBSE D 05 ; OD 05, 15]
6. State Ampere's circuital law and prove
   this law for a circular path around a
   long current carrying conductor.
    [Himachal 98 ; Haryana 98C, 01]                                Fig. 4.170
7. State Ampere's circuital law. Use this         11. A long straight wire of a circular
   law to find magnetic field due to straight     cross-section of radius V carries a steady
   infinite current.                              current '/'. The current is uniformly
                              [CBSE OD 16]        distributed across the cross-section. Apply
                                                  Ampere's circuital law to calculate the
8. A long solenoid with closely wound             magnetic field at a point ‘r’ in the region for
   turns has n turns, per unit of its length.     (i) r <a and (ii) r > a.
   A steady current 7 flows through this
   solenoid. Use Ampere's circuital law to                                        [CBSE D 10]
   obtain an expression, for the magnetic
                                                  12. Write an expression for force acting
   field, at a point on its axis and close to
                                                  on a charge q moving with a velocity in
   its mid point. Draw its field lines.
     [CBSE D 04. C, 14C]                          the region, where magnetic induction ⃗ is
                                                  uniform. How does the speed change, as
9. (a) How is a toroid different from a           the    charge   moves?     Under    what
   solenoid ?                                     circumstances the force shall be zero?
(b) Use Ampere's circuital law to obtain the      [ISCE 96]
    magnetic field inside a toroid.
                                                  13. (a) Write the expression for the force
(c) Show that in an ideal toroid, the             acting on a particle of mass m and charge
    magnetic field (i) inside the toroid and      q moving with velocity in a magnetic field
    (ii) outside the toroid at any point in the   ⃗ . Under what conditions will it move in (i) a
    open space is zero.                           circular path and (ii) a helical path?
                        [CBSE OD 08, 14C]         [CBSE D 17]
10. ( a) State Ampere's circuital         law,    (b) Show that the kinetic energy of the
    expressing it in the integral form.               particle moving in a magnetic field
                                                      remains constant.
(b) Two long coaxial insulated solenoids,
s and s of equal lengths are wound one            14. Consider themotion of a charged
over the other as shown in the figure. A              particle of mass 'm’ and charge 'q'
steady current "I" flows through the inner            moving with velocity in a magnetic
solenoid ^ to the other end B, which is               field ⃗ .
connected to the outer solenoid S, through
which the same current "I" flows in the           (a) If is perpendicular to ⃗ , show that it
opposite direction so as to come out at end       describes a circular path having angular
A. If n and n are the number of turns per         frequency ω = qB / m
unit length, find the magnitude and               (b) If the velocity    has a component
direction of the net magnetic field at a point    parallel to the magnetic field ⃗ , trace the
Answers
1. Refer answer to Q. 2 on page 4.1.                    page 4.26.
2. Refer answer to Q. 3 on page 4.2.              8. Refer answer to Q. 10 on page 4.23.
3. Refer to solution of Example 17 on             9. Refer answer to Q. 11 on page 4.24.
   page 4.16.
                                                  10.      (a)   ⃗ ⃗⃗⃗
Here              or
                                                  (b) (i) Magnetic field inside solenoid
                                                                    Magnetic field inside
                                                  solenoid                 As the currents in
4. Refer answer to Q. 7 on page 4.12.             the two solenoids are oppositely directed,
   See Fig.        4.25.                          so direction of is opposite to that of Bl. The
                                                  net magnetic field at any inside point along
5. Refer answer to Q. 8 on page 4.13.             the axis,
   See Fig.        4.25 on page 4.14.
                                                  B= B1- B2=μ0(n1-n2)I
6. Refer answer to Q. 9 on page 4.22.
                                                  (ii) Outside the combined system, net
7. Refer to solution of Example 33 on
⃗⃗⃗ carrying current I at a distance   from it        solenoid specifying its polarity and
in a vector form.                                     show that it acts as a bar magnet of
                                                      magnetic    moment     m    =    NIA.
Hence derive the expression for the                       [CBSE OD 13, D 15]
magnetic field due to a current carrying
loop of radius R at a point P distant x from
its centre along the axis of the loop.            8. (a) Using Ampere's circuital law, obtain
(b) Explain how Biot-Savart law enables              the expression for the magnetic field
    one to express the Ampere's circuital            due to a long solenoid at a point inside
    law in the integral form, viz., ⃗ ⃗⃗⃗            the solenoid on its axis.
       , where I is the total current passing     (b) In what respect is a toroid different from
    through the surface.                              a solenoid ? Draw and compare the
                              [CBSE OD 15]            pattern of the magnetic field lines in two
                                                      cases.
5. (a) State Ampere's circuital law
                                                  (c) How is the magnetic field inside a given
connecting the line integral of ⃗ over a              solenoid made strong ?
closed path to the net current crossing the
area bounded by the path.                                                       [CBSE OD 11]
(b) Use Ampere's law to derive the formula        9. Derive a mathematical expression for
    for the magnetic field due to an infinitely      the force acting on a current carrying
    long straight current carrying wire.             straight conductor kept in a magnetic
                                                     field. State the rule used to determine
(c) Explain carefully why the derivation as          the direction of this force. Under what
    in (b) is not valid for magnetic field in a      conditions is this force (i) zero and (ii)
    plane normal to a current-carrying               maximum ? [CBSE D 97C, 98]
    straight wire of finite length and passing
    through the midpoint of the axis.             10. Draw a schematic sketch of a
         [CBSE OD 16]                             cyclotron. Explain briefly how it works and
                                                  how it is used to accelerate the charged
6. (a) Show how Biot-Savart law can be            particles.     [CBSE D 08 ; OD 09]
alternatively expressed in the form of
Ampere's circuital law. Use this law to           (i) Show that time period of ions in a
obtain the expression for the magnetic field          cyclotron is independent of both the
inside a solenoid of            length '/',           speed and radius of circular path.
cross-sectional area 'A' having 'N' closely       (ii) What is resonance condition ? How is it
wound turns and carrying a steady current'             used to accelerate the charged
/'. (b) Sketch the magnetic field lines for a          particles ?
finite solenoid. Explain why the field at the
exterior midpoint is weak while at the            11. With the help of a labelled diagram,
interior it is uniform and strong.                    state the underlying principle of a
     [CBSE D 06C, 15C]                                cyclotron. Explain clearly how it works
                                                      to accelerate the charged particles.
7. (a) State Ampere's circuital law. Use              Show that cyclotron frequency is
this law to obtain the expression for the             independent of energy of the particle. Is
magnetic field inside an air cored toroid of          there an upper limit on the energy
average radius ‘r', having 'n' turns per unit         acquired by the particle ? Give reason.
length and carrying a steady current I.
Show that the magnetic field in the open                                  [CBSE Dll, 14, 14C]
space inside and exterior to the toroid is        12. (i) Derive an expression for the force
zero.                                             between two long parallel current carrying
(b) An observer to the left of a solenoid of N    conductors.
    turns each of cross-section area 'A'          (b) Use this expression to define SI unit of
    observes that a steady current / in it            current.
    flows in the clockwise direction. Depict
    the magnetic field lines due to the           (c) A long straight wire AB carries a current
                          Fig. 4.174
13. Derive an expression for the torque
    acting on a loop of N turns, area A,                                 Fig. 4.175
    carrying current 1, when held in a                  15. (a) With the help of a diagram, explain
    magnetic field B. With the help of a                    the principle and working of a moving
    circuit diagram, show how a moving                      coil galvanometer.
    coil galvanometer can be converted
    into an ammeter of given range. Write               (b) What is the importance of a radial
    the necessary mathematical formula.                     magnetic field and how is it produced?
    [CBSE D 04]                                         (c) Why is it necessary to introduce a
14. (a) Two straight long parallel                          cylindrical soft iron core inside the coil
    conductors carry currents l1 and l2 in                  of a galvanometer?
    the same direction. Deduce the                      (d) "Increasing the current sensitivity of a
    expression for the force per unit length                galvanometer may not necessarily
    between them.                                           increase its voltage sensitivity". Justify
Depict the pattern of magnetic field lines                  this statement.
around them.                                                [CBSE D 06, 13C ; OD 14, 14C, 15,16]
Answers
1. (a) Refer answer to Q. 4 on page 4.3.                into two parts : I1 along the smaller part and
                                                        I2 along the larger part of the loop.
(b) Refer answer to Q. 7 on page 4.12.
                                                        Field due to I1 at O,
2. Refer answer to Q. 8 on page 4.13.
                                                            ⃗           normally into the paper.
                  axial                    ⁄
                                                            Field due to I2 at O,
                                                            ⃗           normally out of the paper.
           √                           ⁄
                                           [Put r= 0]
         centre                  ⁄
∴
     √
                    Fig. 4.177
                                                  13. Refer answer to Q. 22 on page 4.53.
The solenoid consists of N loops, each of
area A and carrying a current /. Each loop        14. (a) Refer answer to Q. 21 on page
acts as a magnetic dipole of dipole moment        4.49.
m = IA. As the magnetic moments of all            (b) (i) According to right hand thumb rule,
loops are aligned along the same direction,           the direction of the magnetic moment of
                                [AIIMS 2009]
30. Assertion. An electric field is preferred
    in comparison to magnetic field for                Reason. Opposite currents carrying
    detecting the electron beam in a               wires repel each other.
    television picture tube.
                                                                                 [AIIMS 2015]
Reason. Electric field requires low
  voltage.                                         Answers and Explanations